COVID19 Safety Alert
Skip to main content

Blog Archive

Can My Male Partner’s Infertility Be Corrected? Apr 8th, 2024

Every couple’s fertility journey involves the fertility of both the male and female. Infertility is often a complex problem. Before you place the blame on one partner, it’s important to know what you’re facing.  If we’ve already determined that the male partner has fertility issues, you have a lot of...

What Are My Pre-Treatment Steps of IVF? Mar 13th, 2024

Before you undergo a major procedure such as in-vitro fertilization, or IVF, you’ll want to do a few key things to prepare. Going through IVF is an expensive, time-intensive process, so you want to be as healthy and well-prepared as possible. In-vitro fertilization is rarely a first step on the...

  Preparing for IVF: 6 Essential Tips Feb 9th, 2024

If you’re doing in-vitro fertilization (IVF), you probably have many questions. If you have been trying to get pregnant for a while, you may have reached this step of your infertility journey and are wondering how best to prepare for it.  At its simplest, in-vitro fertilization combines one or more...

Start the New Year Right With a Well-Woman Exam Jan 1st, 2024

You may not think you need to get a well woman exam every year; after all, you only need a Pap smear once every three years if you’re between the ages of 21 and 65. But a whole lot more is included in well woman exams than Pap smears.  Getting...

Chemical Pregnancy Explained: Facts, Myths, and Recovery Tips Dec 6th, 2023

If you’re told that you have a chemical pregnancy, you may have lots of questions. Though this term sounds strangely cold, it just means that your body produced the hormones of pregnancy, but the embryo didn’t get a chance to start developing. You will usually have a miscarriage before the...

Preserving Fertility After a Cancer Diagnosis: Your Comprehensive Guide Nov 1st, 2023

The average age of having your first child has steadily grown. Since 1990, more women have children over the age of 30 than those in their teens or early 20s. While this allows more first-time parents to be well-established in their careers (and often, financially), it also means that some...

Understanding Male Factor Infertility Oct 1st, 2023

Infertility is more common than you might think. Though you may feel alone in your struggles to conceive a child, the truth is that nearly 1 in 7 couples face infertility. Infertility is defined as having unprotected sex on a regular, frequent basis for a year without conceiving, although we...

Eat This, Not That — Infertility Edition Sep 1st, 2023

When you want to conceive a child, you want to maximize your diet. Certain foods can impact your fertility —for better or worse. To make the most of your pursuit of conceiving a baby, you’ll need to learn which foods can give your fertility a boost (as well as which...

Ovulation Induction: Helping Your Body Help Itself Aug 1st, 2023

Sometimes, infertility is a relatively simple matter. If you don’t ovulate regularly, perhaps because you have polycystic ovarian syndrome (PCOS) or another condition that makes it difficult, you may need a little boost. Enter ovulation-inducing drugs such as clomiphene citrate (Clomid®.) Often, ovulation-inducing drugs can make conception possible, especially if we combine them with intrauterine...

The Truth About Stress and Infertility Jul 1st, 2023

If you’re trying to conceive a baby and are experiencing more difficulty than expected, you are probably already tired of hearing people tell you to “just relax.” Simply being told to relax often isn’t perceived as very helpful. Still, it is true that stress does have a link to infertility,...

What to Expect After Automated Semen Analysis Jun 1st, 2023

A basic fertility evaluation involves many complex parts. We have to look at several different factors to help determine the best course of treatment to help you achieve your goal of building your family. Fertility generally includes two major factors: male fertility and female fertility. While we will address all of them during the...

What are Your Chances of Pregnancy After 40? May 1st, 2023

Many people already know that a woman’s fertility sharply decreases beginning in her late 30s. But you may wonder if it’s “too late” to get pregnant after age 40. The answer to that question is complicated and takes into account many factors. Age factors are indeed very real, but sometimes we can...

Amenorrhea: 4 Potential Causes of Missing Periods Apr 1st, 2023

Missed periods might make you feel appreciative, but they’re rarely a good sign for your health. Your body is designed to have regular periods; when you don’t get them, it can indicate potential problems with your health. Sometimes, the reasons why you’re not getting your period are simple; if you’re...

Understanding the Basics of Donor Egg IVF Mar 1st, 2023

You might not be able to conceive a child independently. This could be for several reasons, which we’ll explain in further detail, but, whatever is responsible, your ovaries won’t produce the eggs necessary for conceiving a child. If this is the case, you could benefit from using donor eggs in an in-vitro fertilization...

Serious Health Effects of Menopause and How to Avoid Them Feb 2nd, 2023

Menopause is one of the biggest transitions in a woman’s life. For some, it may be a minor change. But most women experience some degree of discomfort associated with the process, and this is certainly normal. However, while it’s normal, it’s not necessarily harmless. Menopause can have some serious consequences...

7 Important Benefits of Your Annual Well-Woman Exam Jan 1st, 2023

Maybe you haven’t had a well-woman exam in awhile. Life gets busy, and taking care of such matters can easily slip lower on your priority list. It’s no big deal if you wait, right? Actually, attending an annual well-woman exam should earn a high priority on your to-do list. There...

Insulin Resistance and Infertility: Understanding the Connection Dec 1st, 2022

Insulin resistance is a common problem. It’s often linked to obesity, although not always. A lesser known link is to fertility: About 12% of women have difficulty conceiving a child due to insulin resistance.  When you have insulin resistance, you don’t ovulate on a regular basis. This can lead to problems with...

5 Reasons Universal Genetic Testing is Highly Recommended Nov 1st, 2022

Some people might choose to get genetic testing if they know that certain diseases run in their family. But there are plenty of reasons that you should consider getting genetic testing, even if you don’t know of any worrisome family history. Universal genetic testing is always a good idea to help you...

All About Intrauterine Insemination (IUI) Oct 2nd, 2022

In your pursuit of parenthood, you may encounter something called intrauterine insemination, or IUI. It’s one of the first things you may try in getting assistance with conception.  During a natural conception, sperm travels through the vagina to meet with an egg in your uterus. But in this form of artificial...

5 Reasons to Choose a Gestational Carrier Sep 15th, 2022

If you’re facing infertility, you may have a lot of questions about how you’ll still become a parent. Fortunately, you have a lot of options. You can still become a parent through a variety of means. One of your options is choosing a gestational carrier, also known as a surrogate. In this...

Why You Shouldn’t Ignore Severe or Long-Lasting Pelvic Pain Aug 16th, 2022

Pelvic pain is a common condition for many women. Some men are also affected, although it is much less common among men. It can be caused by several factors, so you should seek medical attention if you are experiencing it.. Some pelvic pain is a normal part of the menstrual...

4 Ways to Thrive During Menopause Jul 6th, 2022

Menopause is a big transitional time in a woman’s life. It marks the end of your menstrual cycles and your fertility. And, for many women, it’s also a very uncomfortable time of life. Even though menopause is a biologically normal and natural process, living with the discomforts it presents isn’t something...

Can Your Diet Impact Fertility? Jun 1st, 2022

If you’re trying to conceive, you probably want to make sure that everything you can control is optimal. One of the biggest factors you may wonder about is your diet. Can your diet really make that much of a difference in your fertility? It turns out that your diet is...

Is Infertility Genetic? May 1st, 2022

Infertility is an unfortunately common problem, affecting an estimated 1 in 5 couples between the ages of 15-49. In general, we consider infertility to be an inability to conceive a child after 12 months of trying, although if you’re over 35 and you haven’t had success after six months of trying, we...

Trying to Get Pregnant? Here are Some Dos and Don’ts Apr 4th, 2022

Fertility may seem complicated, especially if you’ve been trying to conceive for a while. But there are a few things you need to learn about what to do and what not to do when you’re trying to get pregnant. Although you may still need additional help getting pregnant, you want...

Stress and Fertility: Here’s What You Should Know Mar 8th, 2022

If you’re having difficulty conceiving a baby, you probably want to know everything you can about why it’s not working. You can do a lot of things to optimize your fertility and investigate why your fertility isn’t up to par. But one of the easiest things to do is to...

The Link Between Polycystic Ovary Syndrome and Infertility Feb 10th, 2022

If you have polycystic ovary syndrome, or PCOS, you’re far from alone. Polycystic ovary syndrome is the most common endocrine disorder for women of childbearing age. If you’re trying to get pregnant and are having difficulty conceiving, it may be due to PCOS; one-third of women with PCOS also have...

Treatment Options for Male Infertility Jan 1st, 2022

You may think that a problem with infertility lies only with a woman’s reproductive system. But the fact is that in over 35% of cases, male factors contribute to infertility. Being diagnosed with male infertility can be hard to hear. Fortunately, male factor infertility is often treatable.  You’ll likely need...

Can I Still Have a Baby After Cancer Treatment? Dec 8th, 2021

Cancer is the second-leading cause of death in the US. Cancer can be a stressful, frightening experience, and to make things worse, cancer treatments can affect your ability to get pregnant. Pregnancy after cancer can and does happen, but it depends on a number of factors, including the quality of...

I Had My Tubes Tied, but Now I've Changed My Mind: What Are My Options? Nov 1st, 2021

Not everyone feels ready for the responsibility of childcare and childrearing. To prevent an unwanted pregnancy, sterilization is a popular option, with an estimated 47% of married couples choosing a vasectomy or tubal ligation over parenthood. Tubal ligation refers to disabling the fallopian tubes, which prevents eggs and sperm from...

The Importance of Wellness Exams Oct 6th, 2021

It’s no secret that regular check-ups with your medical professional are essential to your health. If you don’t know already, one of the many purposes of wellness exams is preventive care — screenings and testings can help doctors catch illnesses and abnormalities before they create serious problems for you later...

The Difference Between Perimenopause and Menopause Sep 19th, 2021

When most people think about menopause, they imagine hot flashes, mood swings, and weight gain, but interestingly, these symptoms are most often associated with perimenopause. If you’re nearing the next phase of your reproductive life, you might wonder what the difference is between perimenopause and menopause. As a board-certified OB/GYN,...

How Age Affects a Woman’s Fertility Aug 16th, 2021

If you’re having a difficult time conceiving after a certain age, you’re not alone. Across the human race, everyone’s fertility declines with age. The natural cycle of aging affects us all, with some being affected differently than others. Having a family can feel like part of that natural cycle, but...

Understanding the IVF Process Jul 7th, 2021

Conceiving naturally is always portrayed as ideal, but if you’re struggling to get pregnant, there are solutions available. You may have already tried a number of fertility treatments, from the herbal and holistic to the medicinal. If other treatments haven’t helped you start or continue the family you’ve been waiting...

Four Lifestyle Changes to Promote Fertility Jun 8th, 2021

We all know the cornerstones of a healthy lifestyle: a nutritious, balanced diet with plenty of water, regular exercise, and a good night’s sleep every night. Your overall health impacts your fertility, so taking care of yourself with these good habits is at the top of the list if you’re...

Is It Possible to Get Pregnant When You Have Endometriosis? May 17th, 2021

If you have endometriosis, you’re not alone. This condition affects hundreds of thousands of women worldwide and is one of the most common causes of infertility. Unfortunately, endometriosis is frequently under-diagnosed, and you may be diagnosed incorrectly before your reproductive problems are properly identified. At The Fertility & Gynecology Center-Monterey...

What a Semen Analysis Can Tell Us Apr 6th, 2021

If you’re having a hard time conceiving a child, it might be time to seek medical expertise. This means you and your partner will probably undergo plenty of tests and consultations. The female reproductive system is enormously complex, and many factors can affect your fertility. That’s why so many mistakenly...

Cryopreservation: When You Want to Preserve Your Fertility Mar 3rd, 2021

More and more women in America are having their egg cells frozen. Some want to extend their fertility until they find the right partner to start a family with. Others do it before cancer treatment makes them sterile. It is a high priority for women recently diagnosed with cancer to...

Myths and Facts About Birth Control Feb 16th, 2021

Birth control gives you options. It gives you the opportunity to be sexually active and not become a parent before you want to. There are also lots of choices when it comes to birth control, so there’s something to suit virtually every woman’s needs.  Unfortunately, there are also many myths...

Tips for Living With Menopause Jan 17th, 2021

Technically, menopause is exactly one day. It’s the day 12 months after your last menstrual period. When people talk about menopause, they are often referring to perimenopause.  Perimenopause is the stage of life leading up to menopause. It can last for as long as 10 years and can bring uncomfortable...

What You Should Know Before Considering IVF Dec 6th, 2020

In vitro fertilization (IVF) has been used successfully since 1978. In the last 50 years, more than 5 million babies have been born as a result of IVF. Today, around 350,000 IVF babies are born annually.  Dr. Edward Ramirez, who leads our team at The Fertility & Gynecology Center -...

Is There a Way to Prevent Infertility? Nov 15th, 2020

About 6% of married women ages 15-44 are unable to become pregnant after a year of trying. In roughly 35% of cases, there’s an identifiable problem with both the woman’s and man’s health causing the infertility, and in about 8%, there’s a problem only with the man’s reproductive system.  At...

My Pap Smear Was Abnormal - Now What? Oct 7th, 2020

If you’re a woman aged 21-65, you should probably have a Pap test every three to five years, depending on your exact age and other factors. No one looks forward to a Pap test, but it’s an important tool for detecting and successfully treating cervical cancer.  At the Fertility and...

5 Common Myths About Infertility Sep 21st, 2020

About 10% of couples who want to conceive experience infertility. Sadly, much of the information that you may happen across is simply wrong, which can leave you confused about both the cause of your infertility and your options.  Edward J. Ramirez, MD, at The Fertility & Gynecology Center - Monterey...

Is Preimplantation Genetic Testing? Aug 6th, 2020

From the moment you decide that in vitro fertilization (IVF) is right for your family, you begin working through a long checklist of items. Genetic testing is among the first and possibly most important of the items on that list.  At the Fertility & Gynecology Center - Monterey Bay IVF,...

5 Symptoms of Endometriosis Jul 24th, 2020

Endometriosis is a fairly common disorder that can range from mild to severe. In some cases, it can even affect your fertility or can cause problems with your bladder, intestines, and other organs.  At Fertility & Gynecology Center - Monterey Bay IVF, board-certified OB/GYN Edward J. Ramirez, MD, has helped...

TTC After Surgery For Stage Four Endometriosis Jun 25th, 2020

Question: Hello, I was diagnosed with stage 4 endometriosis in 2011 (26 yrs old) after a laparoscopy found a large endometrioma. I've never had painful periods prior so that diagnosis was surprising to me.I then grew back another large endometrioma and had my 2nd lap in June 2013. I am...

COVID-19 Update and Measures Taken Mar 19th, 2020

Dear patients, Update: As of April 6, 2020 our clinic will be operating on minimum staff. Dr. Ramirez has been deployed with his U.S. Army medical reserve unit to the East Coast to assist in supporting a field hospital. The staff will still be available to answer questions, refill prescriptions and...

44 Year Old Trying IVF For A Baby For The First Time: Can I Use My Own Eggs? May 17th, 2017

Dear Dr. Ramirez, WOW!!!! I am so glad I found your blog “Women’s Health & Fertility“. You have given lots of great information that is very straight forward. Thank you for that. There are many sites out there and the more you read, the more unreasonable they sound. Here is...

State Of The Art Semen Analysis Feb 24th, 2017

Severe male factor infertility, once a barrier to parenthood, has virtually ceased to exist due to breakthroughs in the field of fertility medicine. One of those is ICSI, a procedure done during the in vitro process by our embryologist. The other, though, comes at the very beginning of an infertility evaluation, “semen analysis”....

Pregnancy at 48–Natural or Donor Eggs? Jan 11th, 2017

Question: This question is actually about my mother and not me. My mom is 48 years old. She has 2 grown up children but she wants to have another child. She knows that her chances of getting pregnant naturally are extremely remote and she knows that egg donation is her...

A Holiday Note Of Hope Dec 21st, 2016

We know that the holidays can be stressful, surrounded with so many expectations. There is pressure from family, friends and yourself to celebrate and to enjoy when unfortunately, for the person experiencing infertility, it can be very difficult to do so. Although you cannot make the pain of infertility disappear miraculously,...

What Is A Frozen Embryo Cycle “FET” And Why Should I Consider It? Oct 14th, 2016

At Monterey Bay IVF, we routinely have patients who undergo a Frozen Embryo Cycle or FET. How this happens and why: For example, while undergoing In Vitro Fertilization (IVF), a predetermined number of quality embryos are normally transferred directly to the woman’s uterus in a fresh cycle. If there were...

Egg Maturity and HCG Trigger: Why Do I Have Immature Eggs? Sep 19th, 2016

QUESTION: I’m 39 years old and have done 3 IVFs. During the first two IVFs, half of my eggs were immature. I had 25 retrieved (age 37) the first time, and 16 retrieved (age 38) the next time with 50 percent maturity in both cases. However, I just had an...

Three IVF Cycles With Immature Eggs: PCOS and HCG Trigger? Sep 19th, 2016

QUESTION: Dear Dr. Ramirez, I'm 39 years old and have done 3 IVFs (in vitro fertilization cycles). During the first two IVFs, half of my eggs were immature. I had 25 retrieved (age 37) the first time, and 16 retrieved (age 38) the next time with 50 percent maturity in...

Ovulation Induction and Exercise Aug 2nd, 2016

Question: Hi Dr. Ramirez, My husband and I are currently having Ovulation Induction (OI) because I have secondary amenorrhea and don’t ovulate by myself. We’ve just been through our third cycle, which (like the other two) resulted in ovulation but no baby! I go for long brisk walks every day (5 miles),...

Dr. Ramirez’ Medical Mission In The Dominican Republic May 25th, 2016

Dr. Ramirez is currently serving with his Army Reserve unit, 352nd Combat Support Hospital, in the Dominican Republic as Chief Medical Officer. As a Lt. Colonel in the reserve medical corps, he is an integral part of the U.S. Southern Command joint task force humanitarian civic and medical mission “Beyond the...

Can I get pregnant again at 46? May 2nd, 2016

Hi Doctor, I recently visited a reproductive endocrinologist because I was having issues with my period (heavy clotty periods).  They did an sonohysterogram and found a 2 cm uterine polyp.  I am due to have that removed in May.  However, they also did blood work to determine how far away...

How to survive the holidays when you have infertility…. Dec 27th, 2015

An excellent article, “Coping with the Holidays…again.” was published by Resolve back in 2009. Written by Linda Cimarusti, it still rings true… Coping with the Holidays — Again By Linda J. Cimarusti, PhDPublished in Resolve, for the journey and beyond, Winter 2009 It’s that time of year again—filled with cheer, shopping, baking, and...

#IVF4VETS Monterey Bay IVF is proud to offer assistance for qualifying Veterans Nov 12th, 2015

“The men and women in our armed forces go where our nation’s leaders direct them, serve in difficult and often dangerous conditions and do so with admirable dedication and professionalism. As a nation, we owe them our freedom. When they are wounded serving their country, we have an obligation to...

Woman With Endometriosis Failed IVF Cycle: Poor Egg Quality? Age Issue? PGS? Nov 3rd, 2015

Question:I am 38 years old from Los Angeles. I just had a failed IVF cycle because my six embryos arrested on Day 5. On Day 3, five were Grade A and one was Grade B. They were 10, 8 and 6 cell. Doctor blames my age for the embryos arresting and basically...

Is There Any Way To Improve Egg Quality? Nov 3rd, 2015

Hello Dr Ramirez. I am writing to you from Nebraska.  I have had 2 fresh IVF cycles and 2 FET cycles.  Only one pregnancy  – from the first IVF 5 years ago.  We have been trying to have a baby through various methods for over 14 years – with only one...

History of Miscarriages, Now 9 Weeks Pregnant: Continue Progesterone Supplement (Crinone) ? Aug 17th, 2015

Question: I'm from the U.S. After a long struggle with secondary infertility and 2 miscarriages, I am pregnant again, 9 weeks along. I'm on supplemental progesterone, Crinone 8% once a day. When can I feel okay about stopping the Crinone? I was supposed to see my doctor in 2 days, but he...

Conceiving After 45: IVF With Your Own Eggs Or Donor Eggs? Jul 14th, 2015

Question: Dr Ramirez, What are the chances of a 45 year old woman conceiving using IVF with her own eggs? Would it be worth trying or would you recommend using donor eggs? A. from the UK Answer: Hello A. from the UK, There are always exceptions to the rule, however, the...

“How In Vitro Fertilization Works” Jul 14th, 2015

Infertility affects 1 in 8 couples worldwide. But in the last 40 years, more than 5 million babies have been born using in vitro fertilization (IVF). How does it work? Nassim Assefi and Brian A. Levine detail the science behind making a baby in a lab.

"How In Vitro Fertilization Works" Video from TED-Ed Jul 13th, 2015

Dear Readers,I recently found this nicely animated video on In Vitro Fertilization created on the new TED-Ed platform. You might find this a good way to not only inform yourself but also friends and family who might have trouble understanding the fertilization process. "Infertility affects 1 in 8 couples worldwide. But in the...

NIAW-National Infertility Awareness Week Apr 23rd, 2015

Every year Resolve, the national infertility association, sponsors a week of advocacy called NIAW across the nation. There are many ways that one can participate, from the “Walk for Hope” to actually lobbying in Washington D.C. for legislation that would aid people with infertility. This year RESOLVE is urging the infertility community...

Secondary Infertility…what test should I have done? Feb 17th, 2015

Hi Dr. Ramirez! I have a beautiful two year old daughter, but I can’t seem to carry another baby. I had a chemical pregnancy earlier this year followed by a blighted ovum. Since then, I have had a recurrent loss panel run. Everything came back normal but a positive ANA result. My...

Submucous Fibroids: Do I Need A Hysterectomy? Dec 12th, 2014

Question: Hello Dr. Ramirez, I’m writing from Mexico. I’m 33 years old, never been pregnant and have just been diagnosed with fibroids. This year I’ve had heavy abnormal bleeding with large clots. First doctor i saw prescribed only birth control as he thought it was only a hormonal imbalance. Months...

Upcoming Frozen Embryo Transfer #4: Do I Transfer 1, 2 or 3???? Oct 1st, 2014

Question: Hello, I have a son via FET. I have now had three cycles of FET total. The first did not work, the second we got my son, and this last one worked- however I had a miscarriage at 6.5 weeks pregnant. I now have 4 embryos left frozen and am...

Failed IVF: Empty Follicle…Should I Try Again? Aug 22nd, 2014

QUESTION: Hi Dr. Ramirez,  I had an failed IVF attempt last week. It was my first attempt. I am 41 yoa. My estrodial was 1950 and I had 5 follicles (14, 16, 20, 20, and 25). No eggs retrieved. My injections included Lupron 20, menopur 250, and Gonal 250. My dr. flushed...

Recurrent Miscarriages: Is It A Hormonal Issue? Aug 21st, 2014

Question: Dear Dr. Ramirez, I am writing from Pennsylvania. In 2006, I had two or three miscarriages.  After that, I went to a fertility clinic and had TSH, prolactin, DRVV, and anti-cardiolipin antibodies tested.  All were normal.  I also had progesterone level checked at the very beginning of one of the pregnancies as...

REQUEST FOR HELP FROM MY READERS Jun 1st, 2014

Dear Readers, It has been a while since I last posted to this blog, and to my readers I apologize.  Things have been quite busy, hectic and stressful this year.  As you know, we are still in a recession here in the U.S. (not officially of course) which has placed...

OPK–How do I read my ovulation predictor kit after my miscarriage? May 1st, 2014

Hi Dr. Ramirez, Thank you very much for your time, I greatly appreciate it. My question is this: I am 30 with 1 child, my husband is 43. I unfortunately had a missed abortion discovered at about 8 weeks and had a d and c 4 weeks ago. The bleeding from...

Post-Lupron & Trying To Conceive Jan 27th, 2014

Hello, I am 24 years old and my husband and I are trying to conceive.   I was diagnosed with endometriosis after a lap on Dec 15, 2007. I have been on two separate treatment cycles of Lupron Depo. The first cycle was in 2011 for 11 months (monthly injection). I...

Could I Be Infertile Or Am I Still Recovering From Surgery For Endo? Jan 23rd, 2014

Question: Hello. I'm a 29 year old female. My husband and I have been trying to conceive for 7 months now. I had a laparoscopy done in June of 2013, due to an ovarian cyst on my right ovary. As the Dr. was doing the procedure, she said that the...

Woman From India TTC Has Blighted Ovum and Recurrant Pregnancy Loss Dec 26th, 2013

Hello Doctor, I am from India and have been TTC (trying to conceive) for last 5 years. I am 29 years old right now. My first miscarriage was when I was 24 years old. It was blighted ovum and was detected on first ultrasound at 8 weeks and d&c (dilatation and curettage) was...

Fertility At 40 After Having Had Children Earlier In Life: Is It Still Good? Dec 7th, 2013

Question: Hello. I just turned 40 years old. I am healthy. I had 1 miscarriage in my early 30s. I waited a few years to conceive after that and conceived two children back to back in the first month of trying so I am hoping my fertility is still good....

I Have Endometriosis With Blocked Tubes & A Previous Ectopic Nov 27th, 2013

Dear Dr. Ramirez, A few years ago, I had an IUD inserted to which my body never adjusted to.  I bled the entire time until it was removed months later to discover that it caused an infection.  This year I became pregnant in January, and in February the ectopic pregnancy...

Poor Responder Needs To Know IVF Is Not All About Numbers: It 's About One Good Embryo Oct 29th, 2013

Question: Hi Dr. Ramirez, My name is A. and I am writing from Michigan. I am 33 years old and have DOR with an AMH of <.16, Hashimoto’s and positive ANA’s. I am on day 10 of stims for IVF #2 and responding poorly compared to our first attempt. I am...

Secondary Infertility: Six Chemical Pregnancies…How to Proceed? Oct 24th, 2013

Hi Dr Ramirez, I have found this site all the way from New Zealand and am hoping you could provide me with some other answers? I already have a daughter who was conceived, carried and delivered easily with no problems at all (that I know of). For the past two years...

After Failing 3 IVF, Reader Has Pregnancy Success After Writing To Me Oct 15th, 2013

Dear Readers, Sometimes I get great news from one of the many couples I help on AllExperts.com and this is one I would like to share with you. Over a year ago I began corresponding with this woman regarding her failed IVF cycles. Her original questions appear right after the good news...

Protect Your Fertility! Sep 28th, 2013

Dear Readers, The American Society of Reproductive Medicine has launched a campaign with the aim of educating women on how to protect their fertility by avoiding certain risk factors. The Society has made available a number of fact sheets, graphics and brochures that are all downloadable on their "Protect Your Fertility" page. At our center...

Trying To Conceive After Stopping The Birth Control Pill Sep 20th, 2013

Hello Dr. Ramirez, I’m a 30-year-old active, healthy female from Texas trying to conceive my first child.  I have been on the birth control pill for 15 years and just stopped taking it two cycles ago in late June.  Based off charting my last two, I have a standard 28-day cycle...

Follicles Too Big In Clomid Ovulation Induction Cycle Sep 16th, 2013

Hello, I am writing from San Diego, CA.  I was on 100mg Clomid on Cycle Day 3-7, then 2mg estradiol on cd 8-12. I went in for an ultrasound to check follicles on cycle day 13.  My RE said that this was most likely a lost cycle because I had...

Similar IVF Protocol But Different Results: Why? Aug 31st, 2013

Question: Hi Dr. Ramirez, I'm back again with a question about my recent IVF (second one). This IVF (in vitro fertilization) cycle we did the same protocol as last time (antagonist) but started off at a higher dose of Gonal-f based on my response last cycle. This cycle we started off...

Is A Tubal Reversal A Good Option For Me? Aug 22nd, 2013

QUESTION: I am a 33 year old mother of two. My children are 11 and nearly 10 years old.  After the birth of my second child in 2003 I had surgery to prevent further pregnancies. (I was in an unhappy marriage and only 23, and couldn't see myself ever getting...

Spaniard Has Questions About IVF Medication Protocol Aug 14th, 2013

Question: Hello Doctor Ramirez, I´m  almost 34 years old, from Spain but living in Mexico City for 8 years now.  I´ve been reading you a lot your blog and you´re questions and answers here. First of all thank you for taking your time, and beeing so patient. We´ve been TTC...

Secondary Amenorrhea: A Description, Not A Diagnosis Aug 9th, 2013

Question: Hello Dr. Ramirez, I'm 20 and 293 pounds. I have had  Secondary Amenorrhea for close to 4 years now. It is apparently being caused by my weight. I'm not planning to TTC for at least another 12 or 13 years. I don't hate babies but it's not just a good...

Should You Worry About That First Six-Week Ultrasound? Jul 15th, 2013

Question: Hello Dr. Ramirez, I’m writing to you from Ontario in Canada. After conceiving via IVF, my wife and I are scheduled for a 7 week ultrasound. At first we were looking forward to this, but after reading around on the internet, we want to cancel this until later when an abdominal...

Woman With Secondary Infertility & Possible Blocked Tube: Misdiagnosed? Surgery? What To Do? Jul 15th, 2013

Question: I had an HSG (hysterosalpingogram) in November. The dye flowed freely through my left tube, which appeared normal and healthy.  However, the dye would not enter my right tube until after the radiologist had me get on my right and then left side. The dye then went into the tube and...

Finally Pregnant After Multiple Miscarriages: "I Am A Nervous Wreck!" Jul 6th, 2013

Hello Dr. Ramirez, I've written you in the past regarding my fertility challenges and your responses have been very encouraging. In my last, I discussed how I'd experienced an early loss in March after our first IVF attempt. You encouraged me to be strong and keep trying that my chances were...

Can I Use Non-Human Derived IVF Medications? Jun 27th, 2013

Question: Dear Dr. Ramirez, I am considering IVF and am more keen on using the non human derived IVF medications. I would be very grateful for  your opinion on Gonal F when compared to Menopur . Also, would you please let me know about the suitability of Ovidrel for IVF....

38 Year Old Has Five Failed Fresh IVF Cycles But Has Frozen Embies: Should She Try FET? Jun 23rd, 2013

Question: Hello - I have been reading you blogs for some time now and am so thankful that you take the time you do with such thoughtful answers. I am 38 and husband is 41. My history is as follows: 2009 wasted time on clomid prescribed by my obstetrician, 2010 saw...

39 Year Old With Recurrent Chemical Pregnancies Jun 17th, 2013

Question: Hello there! I’m writing to you from Florida. I have recently suffered two miscarriages. One in Oct of 2012 and one in March of this year. Both occurred at about two weeks so very early. I guess the term is chemical pregnancy when it is that early. I don't...

36 Yr. Old Has Repeated Implantation Failure With Great Embryos...What's Wrong? May 27th, 2013

Question: Dear Dr, We have just had our 4th failed IVF (in vitro fertilization). Our history.  I am 36, my husband is 39. 1st pregnancy was in 2009 after 3 IuI's (intra uterine insemination) and clomid, but had to terminate at 15 weeks due to large enphaloceale (was a random genetic...

Is A Big IVF Clinic Better Than A Small One? May 19th, 2013

Question: Hello, I am writing from Japan and want to ask that can big, crowded clinics be good or are small ones always good? Which is a worst choice...I'm very confused actually....like the crowded one has all the plus points, it is experienced, and cost is little bit lower than other. The big one...

I Am On Clomid: What Cycle Day Should The IUI Be Done? May 18th, 2013

Dear Doctor, I am on clomid and doing intra uterine insemination. On Cd 5-9, I ovulated. On Cd 18-20 I had an u/s (ultrasound) and cd 13 follies were 8,13,20,20,21. Uterine lining was 6. Will my lining thicken until I ovulate? Should I have an u/s on cd17 instead of...

How To Reduce The Risk Of OHSS May 10th, 2013

Question: Hello, I have been able to get pregnant naturally, but due to my kidney disease was not able to carry to term.  I have about 50 percent kidney function due to mild segmental mesangial sclerorsis.  I'm planning on pursuing surrogacy and would like to know what you would recommend for cycling...

National Infertility Awareness Week 2013....Join The Movement! Apr 22nd, 2013

Dear Readers, This week, in the United States, through the great organization Resolve, we are promoting infertility awareness on many different venues...support groups, chats on Twitter and Facebook and over the blogosphere. April 21st - 27th, 2013 is National Infertility Awareness Week (NIAW).  NIAW is a week dedicated to bringing information about...

Patient Dizzy While On Metformin and Birth Control For PCOS Apr 18th, 2013

Question: Hi doctor, I’m having PCOS (polycystic ovarian syndrome) and for the treatment first I was given Metformin for a month though my sugar level was Normal, and now I’ve been given douluton L (contraceptive) for 21 days. I had just taken it for 2 days and I’m feeling giddy, and have vomiting...

32 Yr. Old Losing Hope After One IUI Miscarriage and One IVF Chemical Pregnancy Apr 8th, 2013

32 Yr. Old Losing Hope After One IUI Miscarriage and One IVF Chemical Pregnancy: I Say Don't Give Up!!! Hello, I don't even know how to begin because my infertility process has been so exhausting. I suppose I have diminished ovarian reserve. My last FSH check was 8.5. My AMH is...

Woman Suspects She Has Endo: Treat The Pain Or Do IVF? Mar 24th, 2013

Question: Hi there. Hubby and I have been TTC (trying to conceive) for almost 4 years now, with no success. We have been to two different REs, the first performed one HSG (hysterosalpingogram) and we underwent two unsuccessful IUIs (intrauterine insemination). The second ran a lot of tests, told me my eggs...

Recent D&C After Miscarriage, Bleeding Abnormally…Can I Still Conceive? Mar 21st, 2013

Question: Dear Doctor Ramirez, I’m 35 with two beautiful kids. On 2/6/13, I underwent a D&C, dilatation and curettage, (approx.  8wks in-missed miscarriage). Bled on and off for two weeks. Approx. 3 days after the bleeding stopped, AF  (menstrual cycle) came for 5 days. I’m now TTC (trying to conceive),...

Young Woman With Endometriosis & PCOS Fails Five Clomid Cycles: Next Step? Mar 14th, 2013

Question: Dear Doctor, Hi, I'm from Minnesota. My husband and I have been trying to conceive since August of 2011. I have endometriosis and PCOS (polycystic ovarian syndrome) since finding out when I was around 17 years old, I'm 25 now. I have had five cycles of Clomid that didn't work. I had...

Infertility Specialist Incompetent: Cancelled IVF Cycle Prematurely Mar 8th, 2013

Dear Doctor, I am 40. I had two missed abortions at 37, both in the ninth week after heart beat was felt. Subsequently I did not conceive and my RE suggested IVF. I have regular 29 day cycle. My Day 3 FSH is normal. My RE put me on Lupron...

Woman With Premature Ovarian Failure Opts For Embryo Adoption: Which Embryos To Choose? Feb 28th, 2013

Question: Dear Dr. Ramirez, I am a 36 year old from Georgia.  I have primary amenorrhea POF (premature ovarian failure). In short always had POF (now called POI) and never had a period on my own.  I guess I have streak ovaries so pregnancy naturally will never be an option. ...

Woman With High FSH & Fibroids Trying To Conceive Feb 28th, 2013

Question: Hello Dr. Ramirez, I am 37 years old with multiple fibroids. My gynecologist gave me Clomid and asked me to test FSH simultaneously and my result came out  22.9, I did ultrasound today and to his surprise there was an egg. He said that I have 3 factors that...

Canadian IVF Cycle Fails: Husband Asks, Try Again? Feb 22nd, 2013

Question: Hello Dr. Ramirez and thank you so much for answering these questions. My wife and I just completed our first IVF (although there was no transfer).  A previous attempt was cancelled due to only a couple follicles (150 Gonal F/150 Menopur).  We had done 2 previous IUIs with Clomid (3 follicles)....

Off The Pill After 12 Years, No Period: Trying To Conceive, What Can I Do? Feb 16th, 2013

Question: Dr. Ramirez, I was on the pill for 12 years, and stopped taking it back in September of 2012 to try and conceive. I did not have a period for over 3 months, so I went to the doctor who gave me a progesterone injection. About a week later,...

Infertility Etiquette Feb 13th, 2013

There are many, many blogs out there written by courageous women who are trying to put into words the very difficult journey they are on…infertility. This particular blog post on infertility etiquette is so well written I thought I would share it in its entirety. It is by Jessah of  “Dreaming of Dimples” who has been trying...

42 Year Old TTC With Endometriomas And Low Antral Follicle Count Feb 4th, 2013

Hi Dr. Ramirez, Not sure what to do.  I met the love of my life later in life and we just started trying to conceive, I am 42 years old.  We really want to have a child, but not so sure about donor eggs .  I have never been pregnant or attempted to...

Woman With Chronic Pelvic Pain From Endometriosis Jan 22nd, 2013

Dear Dr. Ramirez, I am writing from Saint Louis, Missouri. Thank you for taking time out to answer my questions. I will try to be as brief as possible. I am 31 years old and I was diagnosed with endometriosis at age 19. I have had 8 laparoscopies to remove endometriosis,...

What Kind Of Estrogen For Endometrin Priming & Luteal Phase Support? Jan 20th, 2013

Question: Hi there, I'm in Ireland and having egg donation treatment in Spain. I've had several unsuccessful cycles and am now finding that my endometrium is not as thick as it used to be. When my period begins, I take 6mg of Meriestra orally. I was interested to read an earlier...

Recurrent Pregnancy Loss: 5 Miscarriages Since 2009 Jan 11th, 2013

Question: Hello Dr. Ramirez, My husband and I have been trying to conceive since 2007. I have PCOS (polycystic ovarian syndrome) and I have had 5 miscarriages, first one in April 2009 at 10 weeks and the others at 6 weeks. I've also lost a baby due to an incompetent...

Eight Tips On How To Boost Your Fertility Jan 11th, 2013

I often get the question: “What can I do to get pregnant?” or “What can we do to boost our fertility?”.  As an obstetrician/gynecologist who has been specializing in the field of assisted reproduction for over 17 years, I would like to share some tips I have shared with my patients on...

Woman With Asherman’s Syndrome May Need Surrogate Jan 7th, 2013

Dear Dr Edward, I am writing you from Nigeria. I am 28 years old. In 2009, I had a miscarriage at 10 weeks of pregnancy. The doctor performed a D&C (dilatation & curretage) to evacuate the residue from the pregnancy. Since that day till today, I have been having extremely scanty...

"Infertility Nightmare" After TTC For Seven Years & Two Failed IVFs Jan 4th, 2013

QUESTION: Hi, I was very much hoping you could help me with my infertility nightmare! Myself - 30 yrs old, AMH: 3 / FSH 9 / ttc 7 years / diagnosed this year with severe endometriosis mostly around my ovaries. My partner - 39 yrs old - no issues After trying...

Taiwanese Couple TTC Second Child: Three Clomid Cycles So Far Dec 22nd, 2012

Dear Dr. Ramirez, My wife and I have been trying to conceive (TTC) our second child unsuccessfully for the past 3 months now. She is 32 while I am 33. We are both of Asian descent.  She has been to an infertility specialist here in Taipei, Taiwan who prescribed Clomid (50 mg). He...

Sixteen Year Old Does Not Have Her Period Yet: What To Do? Dec 22nd, 2012

Question: Hi, writing from Califonia! My name is E. and I am 16 years old, will be 17 on June 9th, 2013. I'm 5'9" and I am overweight, but not terribly obese or anything. I have always been active, and I have been a swimmer since I was 7. I...

Off Of Birth Control After 5 Years: Period Has Not Returned, Why? Dec 19th, 2012

Hi, I have been on the birth control, known as Sprintec 28, the oral pill for 5 years, and as of this April, I have not taken it. Since I have been off, I have had only one normal period and that was in July. I have had some spotting in October,...

Second IVF Fails Despite Implantation: Thin Lining? Embryo Issue? Dec 10th, 2012

Question: Dear Dr. Ramirez, I am here to seek your advice once again. I just found out my second IVF (in vitro fertilization) attempt finished with a chemical pregnancy. I tested HCG levels at 11dp2dt and it was 19,2 miu/ml (pretty low), and 48h later it was already 4,7 miu/ml. I...

40 Year Old TTC After Termination Of Trisomy Pregnancy Dec 2nd, 2012

Hello, Doctor. I am a 40 YO who has never had any trouble conceiving. I've been pregnant seven times. I had a child when I was 37; all went fine. My husband and I are TTC (trying to conceive). GYN did an AMU (.86) a year ago. He said there was little hope. Nevertheless,...

33 Year Old Fails Multiple IUI and IVF Cycles: What Are Her Options? Nov 29th, 2012

Question: Hello, first, thank you for offering this service. I appreciate the detailed responses and the stories give me hope. I am a 33 year old female from Seattle. I have a beautiful 2 1/2 year old conceived on my third round of iui with femara. My husband and I have...

Woman Has Salpingectomy Of Left Tube: Trying To Conceive Again Nov 28th, 2012

Question: Hi, My name is Lindsay; I’m from Atlanta, Ga. I recently underwent emergency surgery to remove my left Fallopian tube that had ruptured due to an ectopic pregnancy. I lost a severe amount of blood, resulting in receiving 5 blood transfusions. My husband and I have a 20 month...

Disclaimer For The Fertility & Gynecology Center Blog & Website Nov 27th, 2012

November 28, 2012 To my patients, followers and visitors, I am pleased to be writing a blog on women’s health issues. I hope that by doing so I can help people by giving them the information they need from an expert in the field. Please keep in mind, though, that the information...

32 Year Old With Two Failed IVF Cycles With Positive Beta's: Chemical Pregnancies? Nov 20th, 2012

Question: Hi Doctor, I live in Idaho where we only have 1 IVf (in vitro fertilization) clinic, so I don't have the option of a second opinion and can't decide if it's worth the six plus hour drive to find out if anyone else would do anything different so I...

Woman Has Two Failed IUI's With Donor Sperm: Needs An HSG Nov 11th, 2012

Hello, I am a 35-year-old woman writing from Missouri. My husband and I have been trying to conceive for 15 months. After we'd been trying for a year, we went for testing, and my husband was found to have no sperm. We decided to move forward with donor sperm. I had...

High Responder Fails Two IVM and Two IVF Cycles: PCOD & Follicle Maturation Issues Oct 27th, 2012

QUESTION: Hello, During the last year we did two IVM (in vitro maturation) and two IVF (in vitro fertilization) cycles at different clinics at Montreal, Canada. All failed. I'm 39 years old and my husband is 40. My FSH was 11.1 and AMH 2.2 two years ago. The first IVM...

40 Year Old UK Woman Trying IVF After Implantation Failures: Assisted Hatching & PGD? Oct 12th, 2012

PGD QUESTION: dear doctor ramirez, i am so glad to find you on this website and would very much appreciate your help. I will try to give you some background. i have had 3 failed iui, then found problems with my fallopian tubes which led to bilateral salpingectomy, myomectomy, adhesiolysis...

Smoking And Infertility Oct 8th, 2012

Question: Dear Dr. Ramirez, I'm 18 and I just started smoking, but I'm afraid it will effect my ability to have children. I know smoking can cause cancers, but in females, does smoking cause permanent infertility? A friend of mine told me it just causes temporary infertility, and since I'm...

A Step By Step Guide To The IVF Process: Step Seven -- Embryo Transfer Sep 23rd, 2012

Dear Readers: This is the seventh and final part in the series I have begun to help answer what In Vitro Fertilization (IVF) is and how it works with my world-wide Blog audience. What you read here is what I also provide my patients with on a daily basis. I...

A Step By Step Guide To The IVF Process: Step Six -- Embryo Development Sep 3rd, 2012

Blastocyst  Dear Readers: This is the sixth part in the series I have begun to help answer what In Vitro Fertilization (IVF) is and how it works with my world-wide Blog audience. What you read here is what I also provide my patients with on a daily basis. I plan on...

A Step By Step Guide To The IVF Process: Step Five -- Fertilization Aug 16th, 2012

Dear Readers: This is the fifth part in the series I have begun to help answer what In Vitro Fertilization (IVF) is and how it works with my world-wide Blog audience. What you read here is what I also provide my patients with on a daily basis. I plan on...

A Step By Step Guide To The IVF Process: Steps Three And Four--Egg Retrieval Jul 28th, 2012

Dear Readers: This is the fourth part in the series I have begun to help answer what In Vitro Fertilization (IVF) is and how it works with my world-wide Blog audience. What you read here is what I also provide my patients with on a daily basis. I plan on going...

A Step By Step Guide To The IVF Process: Step Two -- Follicle Growth And Egg Maturation Jul 15th, 2012

Dear Readers: This is the third part in the series I have begun to help answer what In Vitro Fertilization (IVF) is and how it works with my world-wide Blog audience. What you read here is what I also provide my patients with on a daily basis. I plan on going into...

A Step By Step Guide To The IVF Process: Step One -- Stimulation Jul 3rd, 2012

Dear Readers, This is the second part in the series I have begun to help answer what In Vitro Fertilization (IVF) is and how it works with my world-wide Blog audience. What you read here is what I also provide my patients with on a daily basis. I plan on going into some...

A Step By Step Guide To The IVF Process: Overview Jun 24th, 2012

Dear Readers, I get many many questions regarding what In Vitro Fertilization (IVF) is and how it works that I thought I would share the information with my world-wide Blog audience. What you read here is what I also provide my patients with on a daily basis. I plan on going into some...

Severe Allergy To Progesterone During IVF Cycles Jun 18th, 2012

QUESTION: Dr. Ramirez, I have a problem with progesterone but my symptoms are not typical of an allergic reaction. My husband and I have done 3 IVF cycles, all failed. They were all chemical pregnancies 1st cycle: The day after the first progesterone in ethyl oleate injection, I developed chills, a high fever,...

3-D Animation Of The IVF Process Jun 13th, 2012

A very nice 3-D animation on In Vitro Fertilization from Schering-Plough (2009).  It details the procedure as well as the fertilization process.

IUTPI: Is It A Better Method Of Intra Uterine Insemination? Jun 8th, 2012

Question: My wife and I are trying for a child for almost a year now and a doctor recommended us this new method of intrauterine insemination, IUTPI (IntraUterine TuboPeritoneal Insemination). I would like to ask you if you have heard about it and of course what is your opinion about...

Recurrent Pregnancy Loss (RPL) Workup Shows MTHFR: What Next? Jun 4th, 2012

Question: Dear Dr. Ramirez, I am 31 years old and have had 3 chemical pregnancies in the past 12 months. They have run a slew of genetic testing and a RPL (recurrent pregnancy loss) workup and all has come back normal with exception to the MTHFR test. I have tested positive for...

Egyptian Fails One Fresh & One Frozen IVF Cycle: Advice On How To Improve Lining Thickness May 31st, 2012

Question: Hi, I am 28 years old, married since 3 years, trying to get pregnant since 2 years. I had ectopic pregnancy since 15 months which ended by right laparoscopic salpingectomy. Then I tried clomid for 3 cycles, HMG induction for 2 cycles and I tried fresh IVF (in vitro fertilization) with long protocol. I...

"The Menopause Map": A New Tool To Help Women Who Suffer From Menopause May 26th, 2012

Although I focus primarily on infertility in this blog, I also wish to expand on other women's issues, such as menopause. A recent article appearing on PBS online, "A New Online Tool Helps Women Track Menopause" caught my eye today. An online interactive tool has been developed by The Endocrine Society called the "Menopause...

Young Canadian With Hydrosalpinx And Endometriosis Complicating IVF Cycle May 19th, 2012

Question: Hello Dr. Ramirez, I am 27 years old and last year stage 4 endometriosis was discovered and I had 2 large cysts removed from both my ovaries, one was 12cm and and the other was 5cm. My left ovary is almost completely gone and both my tubes are not...

Fourth IVF Cycle Ends In A Chemical Pregnancy: What Can Be Done? May 12th, 2012

Question: Hi Dr. Ramirez, I have been on a rollercoaster ride for the past week and I am hoping that you can offer me your opinion. I received a positive beta after my 4th IVF on Monday (11dp3dt). It was 14.3 and my RE said that it was quite low....

IVF Protocol For High FSH May 8th, 2012

Question: Dear Dr. Ramirez, I am 39 years old in 2 weeks and about to undergo my first ivf (in vitro fertilization). I have only one fallopian tube, which an hsg has shown to be blocked, probably by adhesions (my other tube was removed due to damage from extensive adhesions...

35 Year Old Responds Well To Low Dose Clomid IUI Cycle: NOT A Low Responder! May 2nd, 2012

Question: Dr. Ramirez, My husband and I have been infertile for more than 2 years now. We have gone through a wide variety of tests. I have had a polyp removed through operative hysteroscopy and we were asked to try on our own for 3 months afterwards (unsuccessfully). I am...

You Can Do IVF With A Low AMH ! Apr 26th, 2012

Question: Hello Dr. Ramirez, I am 36 and have been unable to conceive for 3 years. The diagnosis we have been given is diminished ovarian reserve. I have tried 3 IUIs (intra uterine insemination) with clomid. Each time I ovulated only 1 egg (I already ovulate on my own). I decided to...

31 Year Old With DOR Advised To Keep Trying: Adjust Protocol To "Mixed" FSH & FSH/LH Apr 20th, 2012

QUESTION: Hi Dr Ramirez I am on my first round of IVF and my day 9 scan showed no follicles on left ovary and 3 tiny follicles on right. I am on 375 menopur which has now been upped to 450. Priot to starting treatment i was on the combined pill...

Australian Undergoing Natural Cycle FET: Timing Is Everything!!! Apr 14th, 2012

Question: Hi Dr Ramirez, I am 32 from Australia. I am about to embark on my first Frozen Empyo Transfer, using my natural cycle. I have one child (5yrs old) who was conceived spontaneosly after removing my endometriosis 3 months earlier. My doctor will be conducting ultrasounds and blood tests...

Israeli Woman Pregnant With IUD In Place: Are There Risks? Apr 11th, 2012

Question: Hello, Sorry for my poor English. I live in Israel, so English is not my mother-tongue :) I had a Mirena placed at late November 2011. Ultrasounds showed that it was placed well. To make thing short: I've got pregnant in Fepuary. My ginecologist didn't succeed to get Mirena out. After...

How Can I Overcome Implantation Failure After Failing Multiple Fresh & Frozen Cycles? Apr 3rd, 2012

Dear Dr Ramirez, I'm a 43 years old female (from Australia) and for the last 2 years have been unsuccessful with IVF (in vitro fertilization) after 3 stimulated cycles and 10 Empyo transfers. I produce a good number of eggs (approx 18) with a stimulated cycle of 300iU/day of Gonal F. This...

Trying To Conceive After Surgery For Cysts & Endometriosis: Do A Clomid Induction Cycle? Mar 29th, 2012

Question: Hi from Wisconsin! My husband and I have been trying to conceive our third child. My youngest will be turning three in a couple of months. They were conceived quickly with no issues. About a year ago, I was advised to have surgery to remove what looked liked a "complex...

Will Removing Blocked Fallopian Tube Help This Patient Conceive? Mar 21st, 2012

Question: Hi again Dr Ramirez, it's K. in NY. I have written in the past about my multiple miscarriages/chemical pregnancies. I tested positive for the MTHFR mutation this past fall. I had a miscarriage at 9 weeks in August, which you believed was most likely due to a virus I...

Conceiving After The Age Of 40: What Are My Chances? Mar 15th, 2012

Question: Hi. I am 43 and began my quest for motherhood about two years ago. I have been on Clomid and Femara and have tried IUI about 5 times. I most recently tried Follistem and IUI. Last month I was on oral contraceptives because of a cyst and returned this...

Woman With History Of Endometriosis Wishes Baby #2 Mar 10th, 2012

March is "Endometriosis Awarness Month" and I thought it would be appropriate to post this recent question from a woman who had endo and succeeded to have her child six months ago. For those of you who wish more information on this reproductive immunologic disease that affects millions worldwide you...

Young IVF Patient Fails Two IVF Cycles: Empty Follicles Mar 4th, 2012

Question: Dear Dr. Ramirez, I am 31 years old, have never been pregnant. My husband and I haver been trying to conceive for about 3 years, tried 5-6 cylces of Femara. I had three failed IUI's and two failed IVF cycles. The first IVF cycle there was one egg but...

Canadian Is Nine Weeks Pregnant, Has Enlarged Yolk Sac And No Fetal Pole: Is There A Problem? Feb 29th, 2012

Question: Hi Dr Ramirez, I was prescribed Clomid this cycle and am now pregnant. I am currently 9 weeks along. I went for an ultrasound at 8 weeks, 1 day pregnant and my measurements indicated that I was 8 weeks, 6 days pregnant. There was no fetal pole found. The yolk sac...

40 Yr Old Wonders: Should I Use Donor Eggs After Failing Five IUIs? Feb 23rd, 2012

Question: Dear Dr. Ramirez, I would like your opinion on whether I should move on to donor eggs. I am 40 yr and I have 5 failed IUI (intra uterine inseminations), 3 of the IUI was with Menopur injections. The last IUI, my RE (reproductive endocrinologist) prescribed 22 vials where I used...

Hope And Encouragement During IVF: Is It Necessary? Feb 16th, 2012

Dear readers old and new, The main purpose of my blog, Facebook entries, Twitter entries and All Experts advice are to give people hope; to urge them to continue their quest to have a child; to give them explanations and to help them in their quest. I and my staff are...

38 Year Old With One Fallopian Tube: Miscarriage With 2nd IVF Feb 11th, 2012

Question: Hello Dr. Ramirez! I am 38 and trying to conceive my 2nd child. I did 2 rounds of IVF at 35 and had a healthy daughter at age 36. We just went through another round of IVF and got pregnant, however it ended up in a miscarriage at 10 weeks. We...

Third Failed IVF Cycle: New Protocol Needed? Compare SART Stats? Feb 7th, 2012

Question: Dr. Ramirez, I just had my 3rd failed IVF cycle and I'm looking for some guidance. A little history: I am 31 have a short luteal phase but PIO and estrace seem to do the trick. Day 3 testing normal. My husband has low morphology. My 1st IVF attempt...

How Can I Have A Year Of The Dragon Baby? Jan 25th, 2012

Question: Dear Doctor, We are a Chinese couple who would like to have a baby this year. We have been trying for many months in the natural way for timing the baby for the Dragon year but we are not successful so far. We are thinking that maybe we can...

34 Year Old With One Tube, Endometriosis, Abnormal ANA: What TTC Strategy Do You Recommend? Jan 21st, 2012

Question: Hi Dr. Ramirez, I am writing to you as I am now desperate with our situation and hoping to get some push from someone who is knowledgeable in this field. I am 34 and my husband is 40. No kids from both sides. We've been TTC for 2.5 years...

Woman Wonders: Natural FET Cycle Vs. Controlled FET Cycle? Jan 12th, 2012

Question: Dr. Ramirez, I have some empyos frozen. I have adenomyois and endo and chronic endometritis diagnosed. Have done antibiotic treatment with uterine lavages and IVs. After depot lupron treatment, is it better to do a natural FET (frozen empyo transfer) or medicated FET. Since it takes about 2-3 months...

Congenital Adrenal Hyperplasia & Infertility Jan 7th, 2012

Question: Dear Dr. Ramirez, Thank-you for reading this message, I greatly appreciate your advice. My husband and I have been trying for a baby for just under 3 years. During the last year we have had 3 cycles of IUI and 3 cycles of IVF all of which have been...

Implantation Problems & Causes Of Chemical Pregnancy Dec 25th, 2011

Question:Hi again, it's K. in NY. I have written to you in the past about my difficulties staying pregnant. I have had 7 chemical pregnancies in the past 18 months and one miscarriage at 9 weeks after using femara (you felt it was probably due to a respiratory virus I...

39 Yr Old TTC With Previous Miscarriage: Clomid Vs. Gonadotropins? Flare Vs. Antagonist Protocol? Dec 17th, 2011

Question:Dear Doctor,I am from India. I am 39. I had two missed abortions at 36 and 37 both in the eighth week and after the heart beat was felt.After leaving a gap of four months I have been trying to conceive naturally for 14 months without any result.Subsequently I started...

Can I Thicken Endometrium With Estrogen? Dec 12th, 2011

Question:Dear Dr. Ramirez,I´m 35 years old (will be 36 in Feb). I have been trying to get pregnant for 2 years (had a miscarriage a year ago). After going to a reproductive clinic, I´ve tried Clomid for 2 cycles with no success, an it really thinned up my endometrium, which...

Did IVF Then Got Shingles: Could It Have Caused BFN? Dec 6th, 2011

Hello,I was 5 days into my 2ww after a second IVF (in vitro fertilization). My first IVF unfortunately was a BFN (big fat negative), when I got my first ever shingles outbreak.Then, my IVF ended up as BFN! Could shingle cause an IVF to fail. I had 4 gradeA embryos transferred. I...

No Period, Retained Cysts & Fibroids, After IVF Cycle Cancelled: 47 Year Old IVF UK Patient Worried Nov 28th, 2011

QUESTION:I am 47yrs. I had my 6th IVF (in vitro fertilization) cycle in September 2011. The drugs used were climara patches10 days before the cycle began, prednisolene 10mg daily, aspirin 81mg daily, bravelle 6 vials every morning, menopur 2 vials every evening later increased to3 vials and antagoni ganirelix acetate...

Surrogate Worried She May Contract Hep B From Transferred Embryo Nov 23rd, 2011

Hello Dr. Ramirez,I'm currently signed up with an agency as a gestational carrier (surrogate). I have been matched with an international couple and was set to have their fertilized embryo transfered into my uterus this month. However I was just informed that the intended father tested positive for Hepatitis B core...

Progesterone After IVF Nov 12th, 2011

Question:My wife had IVF (in vitro fertilization) in Canada. She was prescribed Gonal-F, Repronex and Orgalutran for the stimulation phase. Two blastocysts were transferred at day 5 and yesterday our day-14 serum HCG pregnancy test was positive.We were told by the nurses at the fertility center to stop taking the...

PCOS Patient In India On Clomid: Needs To "Rest" Ovaries & No Ovarian Drilling! Nov 5th, 2011

Question:Hi, I am G. from India. I am 26 yr old and I have PCOS. My LH level (14) is high on day 2. I am trying to conceive now. My gynae suggested me with clomid 100 mg from day 2 to day 7 and hmg 75 on day 2,...

Wondering About Follicle Size And Ovidrel Shot Nov 3rd, 2011

Question:Hi, I am on Gonal-f and my US (ultra-sound) yesterday showed I had 2 follicles measuring at 17mm and 14mm, my doctor told me to take one more shot of 75ui Gonal-f last night and to trigger with ovidrel tonight.Do you think my eggs will be mature enough to ovulate???Thanks!...

Can Lubricants Interfere With Getting Pregnant? Oct 28th, 2011

Question: Hi there. I was wondering if you can tell me if using a lubricant like KY sensitive Jelly can hurt your chances of getting pregnant. My husband and I just started using it a few months ago and we have been trying to conceive. I just read online that...

TTC Patient Needs Aggressive Approach After Laparoscopy For Endometrioma or "Chocolate Cyst" Oct 21st, 2011

Question: Hi, I have been trying to conceive since 1 year. I am 29 yrs old,and a professional with busy working schedules. I recently got myself investigated and found that my FSH levels r 7.09 and LH levels 3.0, AMH levels 3.29. I have undergone 2 ovulation induction cycles which...

IVF Patient Has False Negative On Pregnancy Test: Late Implantation? What Went Wrong? Oct 14th, 2011

Question: Dr.Ramirez, I have written to you in the past and you have always been such a great sounding board. I need help and closure and I am hoping that you can help. In September I went through IVF (in vitro fertilization). On 9/7 I transferred 2 embryos- 1 7cell...

A Little Miracle...Seven Years In The Making Oct 11th, 2011

I want to share a special story with you, my readers, about a couple who went through a recent IVF (in vitro fertilization) cycle with us. This couple had come to us back in 2004 for infertility treatment. After the normal trial of IUI's (intra uterine insemination) did not work,...

Atypia Is NOT An Absolute Indication For Hysterectomy Oct 10th, 2011

Question:Hi Dr. Ramirez,I'm a Canadian, temporarily living in South Africa. Greetings from Pretoria!I'm 44 yrs old, diagnosed with PCOS at age 33, on metformin 500mg 2/day since then. I've got about 45 pounds to lose and have been slowly and steadily losing pounds since May (5 kg). I've never been...

Why Do I Need HCG Injections After Ovulation During IUI Cycle? -1 Oct 5th, 2011

Question:Dr. Ramirez,My husband and I have been trying to start our family for a few years. I have been pregnant and miscarried 3 times, but is has been over a year and a half since my last miscarriage. I am seeing a Reproductive Endocrinologist and their diagnosis for not getting...

Patient On Metformin To Prevent Miscarriage: Is It Necessary? Sep 30th, 2011

Question:HI Dr. Ramirez,Sorry to keep this up about the Metformin, but you have been so helpful in the past...thought I would try your take on this.I talked with my Doc about low dose heparin, and he told me that he does not prescribe this unless tested and confirmed thrombophilia is present, which...

Ovulation Induction With Follistim Keeps Failing & Estradiol Remains Low Sep 25th, 2011

Question:We are currently TTC our 2nd baby. My daughter who is 2 was conceived on our second cycle of follistim75iu. We are currently on our 4th cycle of Follistim 75iu. Each cycle I'm told my estradiol is low and they end up increasing my dose of Follistim to 150 iu...

Secondary Infertility: Decreased Ovarian Reserve And Low Morphology May Be The Culprits Sep 22nd, 2011

Hello,I just turned 33 and I have one beautiful 18 mo little girl who is the love of my life. She was conceived on our 3rd iui using 5mg of Femara. My husband has low morphology (6%) and I have no regular periods. We both exercise / eat as we...

Use Of Prednisone And Lovenox For IVF Cycle With Donor Eggs: How Long? Sep 15th, 2011

Question:Dr. Ramirez:I am a 44-yr old with a history of numerous IVF attempts. Miraculously, cycle 1 (2007) with my own egg (yes, only one egg was retrieved) resulted in a healthy baby. 1 additional IVF attempt (2009) with my own egg - unsuccessful. Subsequently, 4 IVF attempts with two different donors...

Progesterone Supplementation During An IUI Cycle Sep 7th, 2011

Question:Hello Doctor,I have been TTC since the past 15 months, I have irregular menstrual cycles. My husband has no fertility issues. I have been undergoing treatment - clomid, ovidrel followed by IUI (intra uterine insemination) since the past 3 months.My luteal phase is 14 days long. I get a .8/1 degree...

After 11 IUI's, Canadian Fails 1st IVF cycle: Poor Embryos, Bleeding Or Implantation Failure? Sep 3rd, 2011

Question:Dear Dr. Ramirez,I'm writing to you from Toronto, Canada. Thank you in advance for your answer!My husband and I are both 37 years old. I was diagnosed with mild PCOS due to the shape of my ovary (pearl-like follicles) and irregular cycle (28-36 days), and as result was prescribed Metformin....

Woman Has Lost Both Ovaries: Why Does She Still Have A Period? Aug 31st, 2011

Question:Hi,My daughter has lost both ovaries due to cysts. The doctor has put her on Progyluton. We are aware that she cannot get pregnant but the doc says she will still have a bleed. Is this so and if so, how come? S. in Barbados Answer:Hello S. from Barbados,I sincerely...

37 Year Old Fails 3 Clomid Cycles & 2 IUI's: What Should She Do Next? Aug 26th, 2011

Question:Dear Dr. Ramirez: thank you for your great service!I just turned 37 years old and have been ttc for 1 year already. My cycles all my life have been like clockwork (ovulating on day 13 in a 26day cycle). After charting and some blood tests during the early months of...

Secondary Infertility Patient With Seven Miscarriages: Cannot Afford IVF Aug 18th, 2011

Question:Hello-I am 33 and have a healthy 3 1/2 year old daughter who was conceived naturally. I have had 7 miscarriages (1 before my daughter and 6 consecutive since her birth). I have had 4 chemicals, 2 confirmed blighted ovums and 1 xxx69..this one had a heartbeat and all hormones were great,...

Right After A Cancelled IVF Cycle, Try Naturally Or IUI? Aug 13th, 2011

Question:Dear Dr. Ramirez,I write to you from Los Angeles, California. I am 38 and just started an IVF (in vitro fertilization) cycle after two FETs (frozen embryo transfers) that did not take. Those embryos from the FET were from an IVF I did when I was 36 that resulted in...

Previous Ectopic, Left Salpingectomy, Painful HSG: Can I Still Get Pregnant? Aug 6th, 2011

Question:Dear Dr. Ramirez,Approximately 8 years ago, after the birth of my first child, I had an IUD (copper T) inserted. About 6 months later, I lost the thread,and went back to the medical practice for it to be removed and replaced, but was told that losing the thread wasn't a...

New 2011 Study Questions Routine Metformin Use In All PCO Patients Aug 1st, 2011

Dear Readers,A recent study published in the medical journal "Clinical Endocrinology" Frans S., Clinical Endocrinology. [Oxf], 2011; 74:148-151, brings into question the routine use of Metformin in PCO (polycystic ovary) patients. The study showed a small improvement for ovulation but not clearly better than weight loss. It also showed no improvement in...

40 Yr Old German Woman With Great Ovarian Stim Fails IVF: Likely Abnormal Embryos, Try Again! Jul 26th, 2011

Question:Dear Dr. Ramirez,My name is A. from Germany, 40 years old, never pregnant before. I had 2 failed IVF this year (1 in March, 1 in June). My FSH=10.6, AMH=0.8. progesterone level is normal.The 1.IVF had 6 eggs (2 embryos were transferred: 1 grade 1 and 1 grade 2) and 2.IVF had...

Calendar Method Vs. OPK? It's All In The Timing... Jul 23rd, 2011

Question:So my husband and I are trying for our first child. We've trying for about 3 months with no luck yet. i was thinking of trying the ovulation test but i'm not sure if its worth it. I've read online that they can help but i figure it would never...

Failed IVF Cycle With Drop in Estrogen & Progesterone Levels & PCO Type Response Jul 20th, 2011

Failed IVF Cycle With Drop in Estrogen & Progesterone Levels & PCO Type Response: Might Benefit From Adjustment In Protocol Question: HI, I have a question regarding spotting 5 days post transfer with full period bleed on 6 days post transfer. Here is our history:-I am 32 and my husband...

Inadequate Luteal Phase Support In IVF Cycle Can Cause Bleeding & Failure Jul 15th, 2011

Hello Dr. Edward,Thank you in advance for your answer! I'm from Paraguay. I just went through my second IVF-ICSI and both times I started bleeding 7-8 days after day 3 transfer, just when implantation should be occurring. Do I have an implantation problem? Are there any tests I should do?...

Jamaican Woman Doing 1st IVF Confused About Meds: Possible PCOS Misdiagnosis Jul 6th, 2011

Question: I am about to do my first ivf with a clinic and I got a lsit of medications that I will have to use/purchase. I am a litte uncertain about one of them - menopur. The list has over 40 vials of 75iu. I am certain that this is a mistake. I will...

4 Weeks Pregnant, Hypothyroid, At 43 After Clomid Cycle: How Are My Levels?What To Do If I Miscarry? Jul 3rd, 2011

Question:Hello, I’m in California.I just found out I’m pregnant, about 4weeks 4days. I had my first beta; it was only 18 at 14 dpo and 24 at 15 dpo. I took another yesterday at 17 dpo, but I won’t know the results until tomorrow. Either way, a very poor prognosis...

IVF After A Myomectomy & Low Ovarian Reserve Jun 28th, 2011

Question:Hello, Dr. Ramirez, D. from VA here! I wrote to you a few weeks ago concerning my RE wanting to freeze my eggs, give me a myomectomy, then do implantation. You didn't understand why he wanted to do things in that order, and neither did I. He thought that my...

40 Yr Old Austrian Doing 2nd IVF: Use PICSI Or "Embryo Glue"? Antibiotics OK? Jun 8th, 2011

QUESTION:Dear Dr. Ramirez, my name is J. from Austria. I am 40 years old, have a failed 1.IVF (in vitro fertilization) this year (6 eggs, 4 fertilized, only 2 were transferred, because the other 2 arrested development before the transfer) , FSH of 10.6 mIU/mL, AMH of 0.8 ng/mL. My...

Canadian With Recurring Ovarian Cysts And Thick Endometrial Lining Jun 3rd, 2011

QUESTION:I'm 28 years old and have recently been referred to an OBGYN for recurring ovarian cysts. My most recent pelvic ultrasound suggests that I may also have a slightly thicker than usual uterine lining. My new OBGYN has prescribed a birth control pill (Alesse-21) for paroxysmal but intense pain associated with the...

UAE Patient Tested For Infertility: Clomid or Tamoxifen? May 30th, 2011

QUESTION: Hi Doctor,This is S. from UAE. I would like to have your expert opinion in my case. My infertility workup showed multiple cysts in both ovaries but my hormonal work up was all within range. In addition, I have never missed a period ever in life, my cycles are...

Clomid Protocol In Depth: Dosage, Specific Indications & Period Of Use May 23rd, 2011

Hello Doctor,Your willingness to answer fertility questions is admirable. Let me say, "Thank you!"I've been trying to find specific information on your blog regarding clomid protocol and I simply cannot find what I'm looking for. I do find questions and answers regarding clomid, but not specific information on how and when...

Canadian Needs To Know: Frequent Blood Draws Are NOT Necessary For Clomid Induction Cycle May 20th, 2011

Hello Dr. Ramirez,I'm currently undergoing cycle monitoring and my RE has not yet been able to pinpoint a reason for our 18 months of infertility (hubby checked out okay and my initial tests - ultrasounds, blood work, saline hysterosonogram - show no problems other than possibly my age being a...

The Wrong Way To Do A Clomid Induction Cycle May 18th, 2011

Question: Hi. My Doctor prescribed me provera to induce a period and then clomid to induce ovulation. She told me that monthly I am supposed to take provera from the first of the month through the tenth of the month and then once my period comes i'm supposed to take...

The Family Act of 2011 Needs Your Support! May 14th, 2011

Dear Readers and Followers in the U.S.A.,I am taking the time this morning to blog about an important piece of legislation for Infertility patients that needs a co-sponsor in the Senate. This legislation was introduced by Senator Gillibrand of New York and proposes a tax credit for those who undergo...

Mini-IVF In A Woman Over 40 Years Old May 9th, 2011

Question:I am from Canada and will turn 42 in a few weeks. I am trying to conceive my second baby after already having a baby boy with a previous IVF cycle. My first cycle for baby number two was unsuccessful. It consisted of Lupron from day 21 then Gonal F...

40 Year Old IVF Patient In Vietnam On Low Protocol Fails First Cycle: Has Many Questions, Concerns May 4th, 2011

Question:Dear Dr. Ramirez, My name is A. from Vietnam, 40 years old by end of March 2011. Just give you some information about me regarding Infertility/IVF. My menstruation cycles are different every month: 28 days in Feb, 26 days in March and 31 days in April. So, average: 26 days....

How To Interpret Positive BHCG Levels After IVF: When Will You Know For Sure? Apr 30th, 2011

Question: My question is about beta levels, and how to interpret what they indicate.This is my first IVF / first pregnancy, so this is a constant learning process.   1st Beta at 14 dpo (11 days past 3 day transfer) was 158.2nd Beta at 18 dpo was 704.3rd Beta at...

Patient Fails One Fresh, One Frozen IVF Cycle: Will Another FET Work? Apr 28th, 2011

Hello, My name is S. from Boston. I am writing with a question regarding what I am going to be undergoing next week, a second frozen transfer. Just to give you some history, I had a first attempt successful IVF (in vitro fertilization) cycle in 2008 and delivered a healthy...

Cancelled IVF Cycle: Australian Wonders If She Can Risk Conceiving Naturally Apr 21st, 2011

Dear Doctor, I found out today that my IVF (in vitro fertilization) cycle was cancelled due to not enough follicles stimulated. I was day 11 of my cycle and had 6 follicles in total but only 3 measuring over 10 - at 16, 14 and 11. Another was 9. This is my first...

Tubal Reversal Vs. IVF: Which Would Be Better? Asks 38 Yr. Old Mom Who Wants One More Child Apr 17th, 2011

Question: Dear Doctor, I've just turned 38 and I have three children. Right after the birth of my third, I had my tubes tied. Somehow it made sense at the time but now I regret it. My kids are between the ages of 6 and 9 all delivered by c-section,...

37 Year Old TTC With Past History Of Hyperplasia & Endometriosis Is Desperate To Conceive Apr 13th, 2011

QUESTION: Ok, I went to the Gyno in Dec of 2009 because I wasn't getting my period. He sent me for an ultrasound Jan. 2010 and the lining of my uterus was thickened so he did a biopsy which led to my first D&C which was April. I was diagnosed with hyperplasia...

U.K. Woman On Clomid For Five Months, Husband With MFI: Periods Are Shorter Now, Why? Apr 8th, 2011

Question: Hi, I have been taking clomid for 5 months because my husband has a low sperm count. I have noticed my periods becoming less and less the last 2 months I have only bleed for 1 day and that was mostly a black/brown color. My periods normally last for...

Was My IUI Triggered Prematurely? Apr 4th, 2011

Question: Hello, I just had my first IUI (intra uterine insemination) this past Tuesday and although the doctor was not happy with my follicles after Gonal f, he decided that I should trigger anyway with Ovidrel. The ultrasound the day of the trigger showed follicles 16.5mm, 16mm, 15.5mm and also some...

Hydrosalpinx & Fertility:The Difference Between Surgically Caused & Naturally Occurring Hydrosalpinx Mar 30th, 2011

Question: Hi Doctor, I was detected with a dermoid cyst in my right ovary in 2005. I underwent laparoscopy in 2006 and my right ovary was also removed. Post surgery I had normal cycles every month however some months my menstrual flow is less. Last week I had some hormonal tests which...

Diminished Ovarian Reserve, First Time IVF: Low Estradiol (E2) Levels During IVF Stim Cycle Mar 25th, 2011

Question: K. from NYC: So, I took your advice and immediately wasted no time, and moved directly to IVF (in vitro fertilization). Dx: DOR (diminished ovarian reserve) @ age 35, one previous live birth 2 years ago, natural conception, (baby came from left ovary) after 12 months of trying @...

Nicked Uterine Artery During C-Section: Is It Serious? Mar 21st, 2011

Question: Hi there. I am a "mostly" healthy 30 yo female, G2 P2, with chronic HTN, obesity, and history of 1 run of VTAC 5 years ago which was deemed idiopathic in nature. History of pre-e w/baby #1... induced labor, emergent c-section due to fetal decels and failure to progress....

Young Canadian With Endometriosis: Will Have Six Month Window For TTC Post Surgery Mar 20th, 2011

Question: Hi, I’m from Calgary, Alberta in Canada. I am 33 and my husband is 30, we are both healthy and we’ve been TTC unsuccessfully for 19 cycles. I have never been pregnant. We have been tested at the fertility clinic and told that there is no obvious reason why...

Indian Woman TTC For 3 Years, Has Endo: Danazole Or Lupron Therapy? Mar 17th, 2011

Question: I am tryiong to concieve last 3 years. I did on 6 January 2011 report as follows: H-Lap, D&C/ PT(HPE) done Hysteroscopy--Ut cavity regular canal narmal B/L ostia seen. Laproscopy-- Ut normal size. retroverted congestion, Small Seedling fibroid, both ovaries are normal size and healthy. Both Tubes Patents and...

An Indiscretion During An IVF Cycle Mar 12th, 2011

Dear Readers,From time to time I receive letters from women who have no where else to turn for advice. Some months ago I received this letter. In this case, an indiscretion is causing this woman a great deal of anxiety and her positive IVF cycle result hinges on my being able...

Grading Of Embryos At Blastocyst: How Does It Reflect Implantation Rates? Mar 9th, 2011

Question:My husband and I recently went through our first IVF. We have been trying for 2.5 years to get pregnant and have done 6 failed IUIs. We have stage 2 endometriosis that was cleared out by a laparoscopy last Fall. Aside from that we are unexplained infertility. We retrieved 10...

After Failing IVF Three Times 45 Yr Old Wonders, Higher Stim or Lower Stim? Use BCP? Mar 4th, 2011

After Failing IVF Three Times 45 Yr Old Wonders, Higher Stim or Lower Stim? Use BCP?: I Recommend A High Stim Mixed Protocol & BCP Question:Hi Dr. Ramirez,I am 44 turning 45 this June. I have had 3 failed IVFs - 1 didn't go beyond retrieval because I pre-ovulated so...

Secondary Infertility With Miscarriage And Possible Luteal Phase Defect: Is Clomid OK? Feb 25th, 2011

QUESTION: Hi Dr. Ramirez. Here I am writing again. I previously emailed you, I had a miscarriage in Feb 2010, at about 7 weeks. I have a 4 year old and a 3 year old, conceived the first month I tried with both. I also got pregnant the first time...

HCG Trigger Shot Must Be Timed With Ultrasound Surveillance NOT With Calendar Dating When Doing IUI Feb 19th, 2011

Question: I am writing from Hawaii with several questions. We will be undergoing our first IUI soon. The background is the following: I am 36 years old with no fertility factors. We decided to seek out fertility help because my husband, also 36, has an ejaculatory issue. I have been...

Doctor Cancels IVF Cycle Despite Follies Being Present Feb 14th, 2011

Doctor Cancels IVF Cycle Despite Follies Being Present: Can Patient Go On To Do A "Natural" IVF Cycle? HI-This is a pretty simple question - started IVF cycle with micro-dose lupron after 10 days on the pill (for the dr.s convenience) Follistem and menopur. I stimmed for 4 days then cancelled...

Alaskan Worried After First Miscarriage: Was The Cause Low Progesterone? No! Feb 12th, 2011

Question: Hello, I am writing to you from a remote community in Alaska, We have a clinic and Doctors here but, no hospital or specialists, so I am glad to have found your site and hope to receive an answer to my concern. I suffered a miscarriage on Jan 10th...

42 yo U.K. Woman, SWF, Low FSH, Low LH, High TSH, No Periods For 16 Years: What Is Wrong? Feb 6th, 2011

42 yo U.K. Woman, SWF, Low FSH, Low LH, High TSH, No Periods For 16 Years: What Is Wrong? What Are Her Reproductive Options? Question: Dear Doctor, I am a single, white female, aged 42. I have high prolactin and started having no periods at age 26 following cessation of...

Canadian TTC Wants To Know When Is The Best Time To Use A Pregnancy Test Kit Feb 2nd, 2011

Question: My husband and I are trying to get pregnant with our second baby, I got pregnant very easily with my now three year old son. As of right now I am 8 days post ovulation. I have been using 10 miu pregnancy tests which have all come up negative...

40 Year Old U.K. Woman With One Miscarriage Feels Time Is Running Out For Her Jan 27th, 2011

40 Year Old U.K. Woman With One Miscarriage Feels Time Is Running Out For Her: More IUI's, More IVF Or Donor Eggs? This post concerns a woman in England who has written to me several times regarding her infertility journey. I would like to publish the entire correspondence for those...

Ovarian Follicle Size Is Key When Planning For Trigger Shot Jan 24th, 2011

Question: Dear Doctor Ramirez, I had 3 follicles when I received the trigger shot. 30mm 24mm and not sure what size the 3rd one was. Is the 30mm follicle a good size to have a mature egg to be released? Thanks, G. from the U.S. Answer: Hello G. from the...

Alternative Lifestyle Couple Trying To Conceive: Husband Is Transgender Jan 21st, 2011

Question: I just want to know how to go about finding a sperm donor that doesn't cost hundreds of dollars. My wife and I are newlyweds who have settled into our lives together well and are wanting to have a child. Unfortunately I cannot get my wife pregnant (I am...

45 Year Old Woman With FSH Of 13, Fails Four IUI Cycles Jan 18th, 2011

45 Year Old Woman With FSH Of 13, Fails Four IUI Cycles: Go To IVF With Her Own Eggs Or With Donor Eggs? Question: Hello Dr. Ramirez,I'm writing to you from Texas. I just turned 45 and in the past 4 months I have gone through 4 IUI cycles (3...

Norwegian Patient With Arcuate Uterus & PCOS Wants To Know: Is IUI A Good Option? Jan 15th, 2011

Question: Hi, I wrote before. I have an arcuate uterus and polycystic ovaries. My hormonal results were fine as well as my test for cervical cancer. My husband and I have decided to go for artificial insemination since we have been trying to have a baby for 4 years. I have...

39 Year Old IVF Patient, DH Severe Male Factor, Surprised By Treatment Protocol Jan 14th, 2011

39 Year Old IVF Patient, DH Severe Male Factor, Surprised By Treatment Protocol: May Also Have PCOS & Will Need Med Adjustment Question:  Can you help figure out what the next step should be? I'm 39 but have day 3 tests of 6.4 FSH, 53 estradiol and 25 antral follicles....

Donor Eggs and Surrogacy: The Possibility Of A Healthy Pregnancy & A Healthy Child Jan 8th, 2011

Emily Dickinson said: "Dwell In Possibility". I thought I would start the New Year with a blog post that centers on possibilities. I believe that everyone should leave themselves open to the wide array of options that are available to women and men who are struggling with their family building...

Young Woman TTC Is Concerned: Has Abundant Cervical Mucous & Irregular Cycles Dec 27th, 2010

Question: Hello, I am writing you from Alberta, Canada. I have been doing many searches on the internet to find an answer to my question with no success. I hope that you might be able to help me. So, I went off BCP (birth control pill) in March 2010 to...

Young UK Woman Trying To Conceive Has Irregular Cycles, Prior D&C: What Could Be The Problem? Dec 21st, 2010

Question: Good evening, Dr. Ramirez. Thank you for taking the time to read this email. I am concerned about my periods. Nearly two years ago, I made the difficult choice to have an abortion. Not wishing to go through that again, I chose to have the Depo shot for six months after....

UK Patient Has PCOS, On Clomid For Ovulation Dec 15th, 2010

UK Patient Has PCOS, On Clomid For Ovulation: After First Miscarriage, Should She Continue With Clomid Or Start Injectables? QUESTION: I was put on 50mg Clomid due to pcos and long cycles (42-43 days). Right away, my cycles reduced to between 26-32 days. I got pregnant on my 3rd cycle...

37 Yr Old Failed Two IVF Cycles, Has Frozen Embies And Husband With Slow Swimmers Dec 10th, 2010

37 Yr Old Failed Two IVF Cycles, Has Frozen Embies And Husband With Slow Swimmers: What Would You Suggest? Question: I just failed my second IVF (in vitro fertilization) procedure. I am 37 years old living in the Bay Area. My husband has slow swimmers and not great morphology either. We used ICSI (intra cytoplasmic sperm injection)...

Patient With Short Luteal Phase Has Spotting With Progesterone Suppositories, Is This Normal? Dec 7th, 2010

QUESTION: Dr. Ramirez, I am hoping that you can help answer some questions for me as I am feeling confused and hopeless...I am 30 years old and I am trying to concieve. I was on the birth control pill for 9 years and stopped taking it in June. I have...

Can Progesterone Be Given With Clomid Induction? PCO Patient With 3 Failed Cycles Wants To Know... Dec 3rd, 2010

Question:I'm 31, never pregnant. Dx PCOS, receiving metformin 1500 mg a day. Three cycles with clomid with no ovulation. HSG normal, husband semen analysis OK. Now going through second cycle of IUI (intra uterine insemination), ovulation stimulated with Gonal F and triggered with ovidrel. First cycle progesterone level on day...

37 Yr. Old Malaysian Woman Gets Chemical Pregnancy With First IVF Nov 27th, 2010

37 Yr. Old Malaysian Woman Gets Chemical Pregnancy With First IVF: How Did Her First Cycle Look And What Are Her Chances With The Second? Question: Dear Doc, I would appreciate an opinion on this current IVF cycle I'm on and my previous failed cycle. I'm 37. My period is...

Woman Underwent Medical Miscarriage 8 Months Ago & Had Continuous Spotting, No BFP Yet Nov 26th, 2010

Woman Underwent Medical Miscarriage 8 Months Ago & Had Continuous Spotting, No BFP Yet: Should She Be Concerned? Question: Hello, I am a 29 year old female from Pennsylvania. My husband (also 29) and I started trying to conceive last year. We got pregnant on our 2nd month trying, and were...

41 Yr. Old South African Woman Fails IVF: Needs Higher Stim Protocol, Menopur Dose Too Low Nov 23rd, 2010

Question: Hi Dr Ramirez, I am 41 (42 in March) and have just come through a failed IVF cycle (our 1st attempt). I'm not sure what my FSH levels are but know that I only have 4 and 5 antral follicles left and an AMH of 0.94. I was put...

High Dosage Of Clomid For A "Good Egg": Why Double The Dose? Will It Affect Egg Quality? Nov 19th, 2010

Question: Hi Dr. Ramirez, I am writing from Philadelphia, PA and am 39 years old. I have had 9 pregnancies, 7 miscarriages and 2 live births. Since my last child I have had 4 miscarriages and two of the pregnancies I got pregnant with clomid (100mg a day, days 2-6). I...

42 Year Old UK Woman With Irregular Periods, Hot Flashes: Is She Perimenopausal? Nov 15th, 2010

42 Year Old UK Woman With Irregular Periods, Hot Flashes: Is She Perimenopausal? Can She Still Get Pregnant? Question: I am 42 years old and have had irregular periods for about the last 10 months, also hot flushes, etc. I have had blood tests at my doctors which showed I have...

Prior D&C And Infertility: Woman From Barbados Needs A Thorough Infertility Evaluation Nov 12th, 2010

Question: Good Morning All.....My name is D. and I'm from Barbados in the West Indies. Just to give you a bit of background info on myself and hopefully you can make suggestions for me. I've read the testimonials on the site and I was very touched and moved by the...

Stage IV Endometriosis Patient Trying To Conceive: More Time, More Surgery or IVF? Nov 10th, 2010

Question: I was diagnosed 4 years ago with stage IV endo after having it excised during a lap. I had a Mirena IUD inserted during surgery and just had it removed a few months ago because I am ready to have a baby. I am very regular (26 or 28 days) and ovulation...

43 Yr Old With High FSH Of 30 Asks: Is There A Way To Lower It? Nov 6th, 2010

Question: Dear Dr. Ramirez, First, thank you very much for taking the time to answer so many questions. Your column has been an important part of my education on IVF. I am writing from Cincinnati, OH. I am 43 years old and am considering IVF. I just learnt that my FSH...

When Should An IUD Be Removed In Order To Get Pregnant? Nov 5th, 2010

Question: Hi, I am a pretty healthy 25 year old woman with two children ages 5 and 3. I am writing from Washington, DC. I currently have a copper IUD as birthcontrol. I have had it in for a little more than three years now (since I had my youngest...

U.K. Patient Concerned About ICSI And Husband's Diabetes: Will There Be Abnormal Embryos? Nov 3rd, 2010

Question:Dear Dr. Ramirez, My husband had a semen analysis result and he had Moderate Oligospermia and Severe Asthenospermia (only 4% normal progressive). It has been suggested that we would require ICSI IVF, however my husband also has Type 1 diabetes. I have read that his sperm cells might have DNA damage and...

Four Weeks Post First IVF & Positive BHCG: Not Rising Adequately & Possible Ectopic Oct 29th, 2010

Question: Hi Dr. R. - I am 39 years old and we have male factor infertility. In our first completed IVF cycle we had 27 eggs retrieved, 20 good, 14 fertilized, and 3 - day 5 blasts transferred. I had a very light implantation bleed and a positive Hcg at...

Empty Follicle Syndrome In 40 Year Old Attributed To Age Factor Oct 23rd, 2010

Question: Hello Doctor, I am from Cuba but I live in Miami. I am 40 y/o now. I had my first IVF cycle at 39 that failed, back in July. I started 2nd round in Sept.- Oct. but it was just cancelled due to empty follies. It is possible that...

Marathon Runner Has Amenorrhea And Is Trying To Conceive: May Need Gonadotropins Oct 23rd, 2010

Question: Hi, I am a 25 year old, PhD student and I am a competitive marathoner. My husband and I would like to get pregnant over the next 8 months. I was on the pill until last April, when I went off the pill. I had been on the pill for nine...

Patient Runs Risk Of High Multiples With 7 Mature Follicles In IUI Cycle Oct 20th, 2010

Question: I am on my 7th IUI cycle (a miscarriage on the 4th attempt). This month I have 7 follicles mature and ready to go. I am 36 years old. We would gladly welcome twins or even triplets. What is the risk of higher order? A. From the U.S.A. Answer:...

Trying To Conceive For One Year: Clomid Vs. Laparoscopy? Oct 16th, 2010

(If the blog radio program comes on, you can pause it by going to my Oct. 1st post. I will be keeping the show up for the month of October.) Question: Hi. I've been trying to conceive for about a year and my doctor and I are ready to take...

The Importance Of Choosing A Qualified Infertility Specialist: 41 Year Old Patient Losing Time Oct 15th, 2010

(If the blog radio program turns on, please go to my October 1st post & pause it! I will be keeping the show up for the month of Oct.) Question: I have started evaluation with a RE at the beginning of August. I have so far been diagnosed as having...

Faith Salie Comes Out On Her Choice To Undergo Egg Freezing Oct 15th, 2010

(If the blog radio program comes on, please go to the Oct. 1st blog & pause it. I will be keeping the show up for the month of October.) Dear Readers, For those of you who missed this, Faith Salie, a multi-talented American writer, television and radio host decided to...

Guest Blogger Mindy Berkson: Building The Foundation For Surrogacy Brick By Brick Oct 14th, 2010

(If the blog radio program comes on, please go to the Oct. 1st blog & pause it. I will be keeping the show up for the month of October.) Mindy Berkson is visiting my blog once again with an excellent guest post on surrogacy. For many, surrogacy is the only...

Chances Of Pregnancy After Myomectomy: With Or Without IVF Oct 8th, 2010

(If the blog radio program turns on, go to the Oct. 1st blog post and pause it...I will be keeping the show up for the month of October.) Question: Dear Doctor, Hello. I am writing from Atlanta, GA. I wrote once before (concerning my dermoid - thank you for the...

Genetic Defect In Husband's Sperm Leading To IVF Failure: Screening With CGH Recommended Oct 6th, 2010

(If the blog radio program turns on, go to the Oct. 1st blog post and pause it...I will be keeping the show up for the month of October.) Question: Dear Dr. Ramirez, My husband and I have been infertile for two years. The second year, we began IVF. We have...

Woman In Tasmania Has Done 14 IVF Cycles At One Center And Now Faces An FET: What Advice Can You Giv Oct 4th, 2010

(**If the blog-talk radio starts up, go to the October 1st blog post below & pause it...I will be keeping the show up for the month of October.) Question: Dear Dr Ramirez, Someone wrote a comment on my blog suggesting I ask you. Basically, I was asking what questions to...

PCOS Challenge Radio Show Summary: Everything You Wanted To Know About PCOS, Fertility, Menopause, P Sep 30th, 2010

Dear Readers, On September 29th I was interviewed by Sasha Ottey of PCOS Challenge on her BlogTalkRadio program, for an episode titled: "Ask The Fertility Specialist About PCOS And Your Fertility". I was pleasantly surprised at how quickly the one-hour interview went. It was basically a question/answer format (like my blog )....

37 Yr. Old Australian Woman With Complex Hyperplasia With Atypia Does Not Want Hysterectomy Sep 28th, 2010

37 Yr. Old Australian Woman With Complex Hyperplasia With Atypia Does Not Want Hysterectomy: Wants To Conceive Question: I am a 37 yr old woman who has been trying to get pregnant since I was 16. I have never fallen pregnant that I know of. I have had a Laparoscopy...

"Ask The Fertility Specialist About PCOS & Your Fertility" on PCOS Challenge BlogTalk Radio Show Sep 23rd, 2010

I will be interviewed by Sasha Ottey next week on her hour-long internet radio show and I cordially invite you to tune in to what will be an interesting discussion on PCOS and how it affects women who are trying to conceive. To quote the show's host, Sasha Ottey: "It...

200 Blog Posts!!!! All About Infertility & Women's Health Issues...With More To Come! Sep 22nd, 2010

Well, with the last post I have made it to two hundred blog posts since I began in 2008! I have to admit, I started out rather slow but picked up speed when I fell into the format that I now use. Although I have posted overviews on subjects ranging from PCOS to what...

High Prolactin Levels Can Interfere With A Successful IVF Cycle Sep 19th, 2010

Question: Hiya, I'm 28 years old and had the first cycle of ICSI done a month ago. The reason for ICSI was that my husband has a very low sperm count and motility. My test date was 26th August and I started bleeding on the 24th of Aug. I went...

Canadian Wonders If Progesterone Is Needed for Clomid & IUI Cycles: YES! Sep 19th, 2010

QUESTION: I am trying to conceive and I have had 3 rounds of Clomid (100mg) by itself with no success. I have also now done round 4 with IUI. With the IUI cycle, my doctor told me to take 100mg Progesterone 2xday, intervaginally, and 2mg of estrace 2xday. My question...

Excercise Induced Amenorrhea Leads To Poor Health & Infertility Sep 17th, 2010

QUESTION: I have been over exercising for about 10 years now. I get my period fine on birth control pills but when off birth control, I do not get it. I have been off it now for 6 months because the pills are expensive. Is this harmless not to get...

Woman With Hypogonadotropic Hypogonadism Needs FSH, LH and Hormones To Get Pregnant With IUI Sep 15th, 2010

Question: Dear Sir, I have hypogonadotropic hypogonadism. I am trying to get pregnant and did IUI twice, but did not succeed. I was injected with GMH 150. I had to take around 26 injection to develop my follicles and then had to be injected with HCG for ovulation. My E2...

Woman With Chronic BV Asks: "Will Simply Taking Birth Control Pills, Then Going Off, ... Sep 11th, 2010

Woman With Chronic BV Asks: "Will Simply Taking Birth Control Pills, Then Going Off, Cure My Infertility?" Question: Hello. My partner and I have been together for 3 years and have not been able to conceive. He has a daughter from another relationship. We are pretty sure I'm the problem. I...

"What FET Protocol Do You Use For Difficult PCOS Patients?" UK Patient Asks Sep 10th, 2010

Dear Dr Ramirez, Firstly, thank you so much in advance for taking the time to read my question. Brief History: Dx with PCOS at 17 y/o. HSG clear. My husband has severe m/f, so our only chance of conceiving is through IVF with ICSI. On my fresh cycle in 2008,...

Possible PCO Patient Adjusting IVF Antagonist Protocol For Fear Of OHSS: Decrease Gonal-F Dosage? Sep 10th, 2010

Question: I am about to start my first IVF protocol (today is CD2). I am concerned about the recombinant FSH dosage prescribed and would like your opinion regarding appropriate dosage. I believe I am at higher risk for OHSS for several reasons (described below), however my recent ultrasounds are not...

Positive Beta But I Missed Dose Of Prometrium, Now Spotting: Am I Miscarrying? Sep 6th, 2010

Hello Dr. Ramirez, I am on prometrium 200mg/1morning and 2evening. I forgot evening dose on my 12th dpo. Experienced spotting and pencil line blood in mucus next 3 days. Since I forgot to take the prometruim could I be miscarrying? Today it's 16dpo and my beta came back + but...

The Fertility Chase: PCOS - Polycystic Ovarian Syndrome and Fertility Sep 5th, 2010

Back in May 2010, we were privileged to make a movie with The Fertility Chase which aired on the We Network. This short, 7 minute movie is an intimate look at how PCOS, polycystic ovarian syndrome, has affected two women who happen to be patients of our center. They frankly discuss their...

Woman With Prior Anxiety Disorder Hopes To Use Donor Eggs Despite Being Fertile: What Is Donor IVF & Aug 30th, 2010

Question: Hello Dr. Ramirez, Although I'm fertile I don't want biological children who have my genes. However, my husband and I want our children to be biologically his. Is there a way I could go through IVF with a donor egg despite the fact I'm fertile? If this is possible, please...

In Vitro Fertilization Gives You The Opportunity To Become Pregnant But It Is Never A Guarantee Aug 25th, 2010

In Vitro Fertilization Gives You The Opportunity To Become Pregnant But It Is Never A Guarantee: Perseverance Is Key! Something I have been thinking about as I read through infertility blogs & remarks on Facebook, which I think needs to be clarified and clearly understood by all fertility patients. There...

Infertility Bloggers Offer A Great Support Network! Aug 20th, 2010

Dear Readers: I would like to refer you to some of the blogs that I follow that I consider worthwhile. Some of these ladies are still going through IF, some have finally achieved the pregnancy that they have been hoping for, some are going the adoption route & all are infertility advocates....

36 Year Old With 3 Miscarriages On Prometrim Using OPK : Continue With Calendar Method or Go For An Aug 20th, 2010

Question: Hi, My name is R. I am 36 years old living in Atlanta GA. My husband is 41 we do not have any kids and would like to have a baby. In the past I have had 3 miscarriages. prior to my miscarriages, I had a hystersalpinogram(sp) when I...

Paciente Con Endometriosis Severa Y Fracaso De Implantación FIV Aug 14th, 2010

I recently received a question from a woman in Mexico that addressed the problem of IVF Implantation Failure...I have decided to publish it in Spanish. Many who read this blog are Spanish speakers & need to translate the Q & A's. Since I speak Spanish, I would like to publish...

32 Year Old Has Multiple Miscarriages With Secondary Infertility On Clomid With No Success Aug 13th, 2010

Year Old Has Multiple Miscarriages With Secondary Infertility On Clomid With No Success: Should She See An RE? Question: I am 32 and writing from Texas. We are trying to have our 4th child. In Jan. 2010 I had a miscarriage at 7 weeks and subsequent D&C. It took 2...

Infertility Patient Worried About Side-Effects From Lupron Trigger Shot Aug 11th, 2010

Question: Hello. I am about to undergo a cycle in which we will use Lupron as a trigger. I recently have read frightening stories of women who have had long-term, debilitating physical and cognitive side effects after using Lupron even once (usually to treat medical conditions). I am wondering if using Lupron...

Couple TTC, Husband With Diabetes & 42 Yr Old Wife on Fertomid (Clomid): What Can Be Done? Aug 9th, 2010

Question: Hello. My husband and I have been trying to conceive for over one year. I am 42 yrs old and have one ovary. I have regular menstrual cycles but when the ovulation test was done there was no ovulation. I have started to take Fertomid for the first time....

45 Year Old Danish Couple On Sixth IVF Attempt: Should They Alter Meds & Blastocyst Transfer? Aug 8th, 2010

Question: Dear Dr. Ramirez, We are a 45 year old couple from Denmark on our sixth IVF attempt. The first three attempts produced 5/5/9 eggs with 225-275 units of Gonal-F and a fertility rate of 100%/60%/60% respectively. Eggs for transfer were three 8-cells on try one on day three, two 8-cells and...

Amenorrhea & Secondary Infertility In A 29 Year Old: Needs Evaluation For POF, Thyroid Or Pituitary Jul 23rd, 2010

Hi Dr. Ramirez, I am a 29 yr old healthy female with two daughters, ages 13 & 12. Me and my husband conceived with no problem. I was 16 when we had our first daughter. I started having irregular menstrual cycles when my youngest was 2. We tried to get...

Possible Uterine Abnormality Or Immunologic Disorder Causing Four Miscarriages In 32 Year Old Jul 20th, 2010

Possible Uterine Abnormality Or Immunologic Disorder Causing Four Miscarriages In 32 Year Old: IVF With Surrogate Again? Question: Background: I am 32 as is my husband. We had an ectopic pregnancy 10/2005, 8.5 week miscarriage 3/2006, 10 week miscarriage 8/2006, 9 week miscarriage 2/2007 (genetic testing done - female/chromosomally normal),...

American In The UK Taking Clomid & Has Thin Uterine Lining: Needs A Specialist Jul 19th, 2010

Question: Hi Dr. Ramirez, I will try to keep this as concise as possible. I am very healthy, slim and 29 years old, and have never had issues with my periods. My husband (29 years also) and I conceived on our first try last year, but unfortunately had a missed...

TTC For Seven Years & Dealing With Male Factor Infertility: Do We Do IUI or IVF? Jul 17th, 2010

Question: My hubby and I have been trying to conceive for 7 yrs. One specialist told us only IVF (in vitro fertilization)would work, but a second opinion told us IUI (intra uterine insemination) would work. He has a count of 9mill and motility of 60%. I'm perfectly fine, my eggs...

Australian Woman Has Possible Semen Allergy & Yeast Infection or Thrush Jul 14th, 2010

Question: Ever since my partner and I have been having unprotected sex, I have been having a severe reaction straight after he ejaculates in me. It happens almost immediately with swelling internally and externally, burning, and pain for at least the next two hours. If I wash myself quickly sometimes...

A Positive BHCG But Now Bleeding Two Weeks After Embryo Transfer: Am I Miscarrying? Jul 14th, 2010

Question: Hello. I did an IVF transfer June 24th, was told it was a beautiful transfer of 3 beautiful embryos. This past Tuesday, July 6th, I got a positive result, however doc told me to be cautiously optimistic as it could be a chemical pregnancy because my level was 33. The...

Prometrium Vs. Endometrim For Luteal Phase Support In IVF Jul 12th, 2010

Question: Dear Dr. Ramirez, I've written to you before about luteal phase bleeding. We've tried 3 IUI's, but my husband's morphology has been consistently 0%, so we are moving on to IVF with ICSI. My RE has me on a long protocol (BCP, Synarel, Puregon, Repronex). My main concern is...

Overweight Woman Trying To Conceive Has Irregular Periods And Multiple Miscarriages Jul 11th, 2010

Question: Hi Dr. Ramirez, I read your blog and I love it! I learned a lot reading all your archives, so I wanted to come to you for advice about my situation. I am 24 years old and my husband is 26. When I was 21, I was told that...

Husband & Wife With Different Blood Types: Did Rh Factor Lead To A Fetal Demise? Jun 29th, 2010

Question: Hi, I'm 33 years old my husband is 39. We are a healthy couple. Twice in one year we've gotten pregnant. First pregnancy, very early, I began to have spotting and eventually was diagnosed with an ectopic pregancy, I was given Methotrexate. This past May I was pregnant again,...

Diminished Ovarian Reserve: I Have Failed 3 IVF Cycles, Blighted Ovum -- Where Should I Go Next? Jun 24th, 2010

Question: Dr., I am 34 and have been diagnosed with diminished reserve. I have done three IVF cycles. In the first, we got 5 eggs, after fertilizing all had testing and all found to have some anomoly, none transferred. Second, the clinic let me ovulate...didn't even get to extract eggs....

Male Factor Infertility -- IVF Cycle FET Positive But Ended With Miscarriage: Should I Seek ... Jun 21st, 2010

Male Factor Infertility -- IVF Cycle FET Positive But Ended With Miscarriage: Should I Seek A Second Opinion? Question: Dr. Ramirez, My husband and I are a 29yo healthy couple with male factor infertility. Motility is less than 1%, morphology is about 25-30%, and count is pretty low-normal. There are...

IVF Patient With Positive Beta On Immunoglobulins (IVIG) & Has Questions Jun 17th, 2010

Question: Good day Dr Ramirez, I wrote to you a while ago. We just completed our 3rd IVF (in vitro fertilization cycle) and 2 , 5-day frozen blastocysts were transferred. I had a blood test on day 10 post transfer and the result was positive. I suffer from rheumatoid arthritis. My doctor...

Woman With Stage 4 Endo Worries About Endometrioma: Will Not Interfere With IVF, Can Be Excised ... Jun 15th, 2010

Woman With Stage 4 Endo Worries About Endometrioma: Will Not Interfere With IVF, Can Be Excised At A Later Date Question: I am 28 years old & TTC for a while. Diagnosed with stage 4 endo in Dec. 09. I was told to do 6 months of lupron, then another...